PRE-Mix March 2021 - Sleepy Classes

90
PRE-Mix (Compilations of the Multiple Choice Questions) For the Month Of March 2021 Visit our website www.sleepyclasses.com or our YouTube channel for entire GS Course FREE of cost Also Available: Prelims Crash Course || Prelims Test Series

Transcript of PRE-Mix March 2021 - Sleepy Classes

PRE-Mix (Compilations of the Multiple Choice Questions)

For the Month

Of

March 2021

Visit our website www.sleepyclasses.com or

our YouTube channel for entire GS Course FREE of cost

Also Available: Prelims Crash Course || Prelims Test Series

T.me/SleepyClasses

Table of Contents

1. Geography 1 .............................................................................................................

2. History 9 ....................................................................................................................

3. Polity & Governance 26 ........................................................................................

4. Economy 39 ................................................................................................................

5. Environment 57 .........................................................................................................

6. Science & Technology 75.......................................................................................

www.sleepyclasses.com

Call 6280133177

T.me/SleepyClasses

1. Geography To watch the following questions on YouTube, click on the links given below

• Video 1

• Video 2

1. Arrange the ports from North to South as per their “Geographical Location “

1. Chittagong Port

2. Sittwe Port

3. Kyaukpyu Port

A. 1, 2, 3

B. 1, 3,2

C. 2, 1,3

D. 3, 2,1

Answer: A

Explanation

www.sleepyclasses.com

Call 6280133177Available on App store

and Play Store 1

T.me/SleepyClasses

2. Consider the following statements:

1. Jet streams occur in the Northern Hemisphere only.

2. Only some cyclones develop an eye.

3. The temperature inside the eye of a cyclone is nearly 10 deg C lesser than that of the

surroundings.

Which of the following statements are true ?

A. 1 only

B. 2 and 3 only

C. 2 only

D. 1 and 3 only

Answer: C

Explanation

2Available on App store

and Play Storewww.sleepyclasses.com

Call 6280133177

T.me/SleepyClasses

• The eye temperature may be 10°C warmer or more at an altitude of 12 km than the surrounding

environment, but only 0-2°C warmer at the surface in the tropical cyclone.

3. Recently the term “ Backstop Agreement “ was in news . It was in context of :

A. TPP

B. RCEP

C. BREXIT

D. MERCOSUR

Answer: C

Explanation

www.sleepyclasses.com

Call 6280133177Available on App store

and Play Store 3

T.me/SleepyClasses

4. Boyoma Falls are a system of waterfalls that hold the record as the largest waterfall by volume of

annual flow rate. It is located in which of the following country?

A. Democratic Republic of Congo

B. Nigeria

C. Tanzania

D. Uganda

Answer: A

Explanation

• They form the largest waterfall by volume of annual flow rate in the world, exceeding both the

Niagara Falls and the Iguazu Falls.

5. Which part of the Sun is visible during a Total Solar Eclipse

A. Photosphere

B. Corona

C. Heliosphere

D. None of the above

Answer: B

Explanation

• The corona is the outermost layer of the Sun, starting at about 1300 miles (2100 km) above the solar

surface (the photosphere). The temperature in the corona is 500,000 K (900,000 degrees F, 500,000

degrees C) or more, up to a few million K.

• The corona cannot be seen with the naked eye except during a total solar eclipse, or with the use of a

coronagraph.

6. Which of the following are true for Western Disturbance ?

1. It is an tropical storm originating in the Mediterranean region.

2.  It leads to non-monsoonal precipitation pattern driven by the westerlies.

3.  It is responsible brings sudden winter rain to the northwestern parts of the Indian subcontinent.

A. 1 and 2 only

B. 2 and 3 only

C.  1 and 3 only

D. 1, 2 and 3

Answer: B

4Available on App store

and Play Storewww.sleepyclasses.com

Call 6280133177

T.me/SleepyClasses

Explanation

• A western disturbance is an extra tropical storm

originating in the Mediterranean region that

brings sudden winter rain to the northwestern

parts of the Indian subcontinent.

• It is a non-monsoonal precipitation pattern

driven by the westerlies.

• The moisture in these storms usually originates

over the Mediterranean Sea, the Caspian Sea

and the Black Sea

• Western disturbances are important for the development of the Rabi crop which includes the locally

important staple wheat.

www.sleepyclasses.com

Call 6280133177Available on App store

and Play Store 5

T.me/SleepyClasses

7. What are the factors which may affect “Surface Ocean Salinity” ?

1. Evaporation

2. Precipitation

3. Fresh water flow

4. Wind

5. Ocean Currents

A. 1, 2 3 and 5

B. 1, 2 , 3 and 5

C. 1, 2, 3 and 4

D. All of the above

Answer: D

Explanation

Remember

• The low salinity trend is observed in the Bay of Bengal due to influx of river water by the river Ganga.

6Available on App store

and Play Storewww.sleepyclasses.com

Call 6280133177

T.me/SleepyClasses

• On the contrary, the Arabian Sea shows higher salinity due to high evaporation and low influx of fresh

water.

8. Which of the following statement(s) is/are true?

1. Majority of agricultural land in India is under some form of irrigation than being rain fed.

2. The mean productivity of both irrigated and rain-fed lands is almost the same.

A. 1 only

B. 2 only

C. Both1 and 2

D. Neither 1 nor 2

Answer: D

Explanation

• Irrigated area accounts for nearly 48.8 per cent of the 140 million hectare of agricultural land in India.

The remaining 51.2 per cent is rainfed.

• The mean productivity of rainfed area (71.62 mha) is about 1.1 tonne per ha compared to 2.8 tonne

per hectare of irrigated area.

9. Which of the following is / are the characteristics of Indian coal?

1. High ash content

2. Low Sulphur content

3. Low ash fusion temperature

Select the correct answer using the codes given below:

A. 1 and 2 only

B. 2 only

C. 1 and 3 only

D. 1, 2 and 3

Answer: A

Explanation

• Statement 1 is correct as Indian coal has high ash content.Statement 2 is correct as content of

Sulphur is low in Indian coal.

• Statement 3 is incorrect as Indian coal has high low ash fusion temperature.

• Indian coals like any other gondwana coals are of high ash content which due to the nature of

formation of the coal deposits.

www.sleepyclasses.com

Call 6280133177Available on App store

and Play Store 7

T.me/SleepyClasses

• In general Indian Coals are of drift origin, i.e. the original plant materials, which were later

transformed into coal, were transported by rivers and laid down as fluviatile, lacustrine or deltaic

deposits. As a result the plant material were contaminated with clay and other detritus minerals,

giving rise to high ash content

10.Strait of Gallipoli connects:

A. Sea of Marmara and Aegean Sea

B. Aegean Sea and Ionian Sea

C. Black Sea and Sea of Marmara

D. Caspian Sea and Black Sea

Answer: A

Explanation

8Available on App store

and Play Storewww.sleepyclasses.com

Call 6280133177

T.me/SleepyClasses

2. History To watch the following questions on YouTube, click on the links given below

• Video 1

• Video 2

• Video 3

• Video 4

1. Which of the following are Iltutmish’s contributions?

1. Iqtadari System

2. Tanka

3. Jittal

Select the correct answer using the codes given below.

A. 3 only

B. 1 only

C. 2 and 3 only

D. All of the above

Answer: D

Explanation

• Iltutmish made a significant contribution in giving shape to administrative institution such as iqtas,

army and currency system. ·

• He gave the Sultanate two of its basic coins– the silver ‘Tanka’ and the copper ‘Jittal’. To affect greater

control over the conquered areas Iltutmish granted iqtas (land assignments in lieu of cash salaries) to

his Turkish officers on a large scale.

• The recipients of “iqtas” called the “iqtadars” collected the land revenue from the territories under

them. Out of this they maintained an armed contingent for the service of the state, enforced law and

order and met their own expenses.

• Iltutmish realized the economic potentiality of the Doab and the iqtas were distributed mainly in this

region. This secured for Iltutmish the financial and administrative control over one of the most

prestigious regions of North India

2. Consider the following:

1. Sawar

2. Du-aspah

www.sleepyclasses.com

Call 6280133177Available on App store

and Play Store 9

T.me/SleepyClasses

3. Zat

4. Mansabdari System

Which of the above were introduced by Akbar?

A. 4 only

B. 1, 2 and 4 only

C. 3 and 4 only

D. 1, 3 and 4 only

Answer: D

Explanation

• There is, however, a controversy regarding the Mansabdari system i.e. when it started precisely. From

the available evidence, it appears that this system had been initiated by Akbar (in 1577). Along with

Mansabdari system, Akbar also reformed the revenue system and introduced two new concepts

namely ‘Zat’ and ‘Sawar.’

• Jahangir also introduced a system whereby the selected nobles could be allowed to maintain a larger

quota of troopers, without raising their Zatrank. The system was popular as ‘duaspah’ (a trooper with

two horses) or ‘sih-aspah’ (a trooper with three horses) system.

3. Which of the following events left Ripon totally disillusioned and heartbroken and because of which

he tendered his resignation and left for England?

A. Passage of First Factory Act 1881

B. Illbert Bill passage

C. Non passage of Self government Act

D. William Hunter Commission controversy

Answer: B

Explanation

• According to the system of law, a European could be tried only by a European Judge or a European

Magistrate.

• The disqualification was unjust and it was sought to cast a needless discredit and dishonour upon the

Indian-born members of the judiciary. C.P. Ilbert, Law Member, introduced a bill in 1883 to abolish this

discrimination in judiciary.

• But Europeans opposed this Bill strongly. They even raised a fund of one lakh fifty thousand rupees

and established an organisation called the Defence Association. They also suggested that it was

better to end the English rule in India than to allow the English to be subjected to the Indian Judges

and Magistrates.

• The press in England joined the issue. Hence, Ripon amended the bill to satisfy the English in India and

England.

10Available on App store

and Play Storewww.sleepyclasses.com

Call 6280133177

T.me/SleepyClasses

• The Ilbert Bill controversy helped the cause of Indian nationalism. The Ilbert Bill Controversy is a high

watermark in the history of Indian National Movement. Ripon was totally disillusioned and

heartbroken and he tendered his resignation and left for England.

• The immediate result of this awakening of India was the birth of the Indian National Congress in

1885, the very next year of Ripon‘s departure.

4. Arrange the following states in the correct chronological order of them accepting the British

Subsidiary Alliance system?

1. Hyderabad

2. Awadh

3. Mysore

Select the correct code.

A. 1-2-3

B. 3-2-1

C. 1-3-2

D. 3-1-2

Answer: C

Explanation

• The Indian princes who accepted the subsidiary system were:

✓the Nizam of Hyderabad (September 1798 and 1800),

✓the ruler of Mysore (1799),

✓the ruler of Tanjore (October 1799),

✓the Nawab of Awadh (November 1801),

✓the Peshwa (December 1801),

✓the Bhonsle Raja of Berar (December 1803),

✓the Sindhia (February 1804),

✓the Rajput states of Jodhpur, Jaipur, Macheri, Bundi and the ruler of Bharatpur (1818).

✓The Holkars were the last Maratha confederation to accept the Subsidiary Alliance in 1818.

5. Welby Commission is famous in Modern Indian History for which of the following reasons?

A. Educational reforms

B. Relations between British crown and Indian princely states

C. Police reforms

www.sleepyclasses.com

Call 6280133177Available on App store

and Play Store 11

T.me/SleepyClasses

D. To enquire into the Drain theory

Answer: D

Explanation

• Dadabhai Naoroji in his famous book Poverty and UnBritish Rule in India wrote his Drain Theory. He

showed how India‘s wealth was going away to England in the form of:

✓salaries,

✓savings,

✓pensions,

✓payments to British troops in India and e) profits of the British companies.

• In fact, the British Government was forced to appoint the Welby Commission, with Dadabhai as the

first Indian as its member, to enquire into the matter.

6. Khujli Ghar recently heard in news is associated with which of the following states?

A. Nagaland

B. Uttar Pradesh

C. Bihar

D. West Bengal

Answer: A

Explanation

• Some villages in Nagaland are trying to revive a traditional form of punishment to reduce crime.

• Khujli ghar is a cramped, triangular cage made from the logs of Masang-fung. Masang-fung is a local

tree that causes irritation.

• Social offenders of Naga customary laws dread this punishment due to humiliation within the

community.

• Such itchy cages are referred to as khujli ghar in Nagamese

• The Aos, one of the major tribes of Nagaland, call it Shi-ki (flesh-house).

• The cage is usually placed at a central spot in the village, usually in front of the morung (bachelor’s

dormitory) for the inmate to be in full public view.

• It does not affect the palm but people who make the cages have to be careful.

• Article 371(A) of the Constitution guarantees the preservation of the Naga customary laws.

• The State also funds the customary courts in villages and towns where cases — mostly dealing with

land litigation, money-lending and marital disputes — have a high rate of prompt disposal.

Article 371A

12Available on App store

and Play Storewww.sleepyclasses.com

Call 6280133177

T.me/SleepyClasses

• Parliament cannot legislate in matters of Naga religion or social practices, the Naga customary law

and procedure, administration of civil and criminal justice involving decisions according to Naga

customary law.

• Parliament also cannot intervene in ownership and transfer of land and its resources, without the

concurrence of the Legislative Assembly of the state.

• This provision was inserted in the Constitution after a 16-point agreement between the Centre and

the Naga People’s Convention in 1960, which led to the creation of Nagaland in 1963.

7. Which of the following pairs have been correctly matched?

1. Houthi- Syria

2. Hazara- Iran

3. Cossacks- Ukraine

4. Moor- Sudan

A. 1 and 2 only

B. 2 and 3 only

C. 1, 3 and 4 only

D. 2, 3 and 4 only

Answer: B

Explanation

• Houthis - Yemen

• Cossacks - Ukraine and southern

• Russia Moor - Morrocan

• Hazara - Afghanistan, Pakistan and Iran

8. Based on the following statements identify the temple?

1. The temple is dedicated to Maa Bagulamukhi Devi who is a powerful manifestation of Goddess

Durga.

2. It is believed that Raja Veersen, the then ruler of Dongargarh, in order to appease the Gods for his

true successor, had built the temple 2200 years ago

3. The temple is located near Durg in the state of Chattishgarh.

A. Maa Vindhyachal Vasini Temple

B. Maa Bamleshwari Devi Temple

C. Achru Mata Temple

www.sleepyclasses.com

Call 6280133177Available on App store

and Play Store 13

T.me/SleepyClasses

D. Harsiddhi Durga Temple

Answer: B

Explanation

• Foundation Stone was laid down for development of Maa

Bamleshwari Devi Temple, Dongargarh, Chhattisgarh under

PRASHAD Scheme

• The temple here is dedicated to Maa Bagulamukhi Devi who

is a powerful manifestation of Goddess Durga.

• ‘National Mission on Pilgrimage Rejuvenation and Spiritual,

Heritage Augmentation Drive’ (PRASHAD)

• It is a Central Sector Scheme

• Launched by: Ministry of Tourism in 2014-15

• Objective: Integrated development of identified pilgrimage and heritage destinations

• Aim: Infrastructure development such as entry points (Road, Rail and Water Transport), last mile

connectivity, ATM/ Money exchange, area lighting and illumination with renewable sources, first aid

centers, etc

• Till now, 13 projects have been successfully completed under PRASHAD Scheme.

9. The musical instrument Santoor is a traditional instrument of

A. Uttar Pradesh

B. Odisha

C. Tamil Nadu

D. Jammu and Kashmir

Answer: D

Explanation

• In India, "Santoor" was used as an accompaniment instrument to

the folk music of Kashmir.

• It was a 100-stringed instrument played in a style of music known as

the Sufiana Mausiqi.

• The Sufi mystics used it as an accompaniment to their hymns.

• The original Sanskrit name of Santoor was "Shatha Tantri Veena"

meaning a lute or a stringed instrument that has over hundred strings.

• Santoor is a Persian name to this same instrument "Shatha Tantri Veena" that has references back to

Vedic literature.

14Available on App store

and Play Storewww.sleepyclasses.com

Call 6280133177

T.me/SleepyClasses

10.Navjivan newspaper is associated with which of the following personalities?

A. M.K. Gandhi

B. Sunder Singh Lyallpuri

C. Ganesh Shankar Vidyarthi

D. Tarak Nath Das

Answer: A

Explanation

• Navjivan India is an Indian newspaper published by The Associated Journals Ltd who began

publishing the daily Navjivan on 1 November 1947.

• Earlier, a newspaper called Navjivan was published by Mohandas Gandhi, therefore with his

permission, The Associate Journals started publishing Navjivan.

• Just like National Herald and Qaumi Awaz, Navajivan was also started with the intention to promote

the principles of Mahatma Gandhi's freedom movement and Jawaharlal Nehru’s vision of modern

India.

11.Which of the following statements are correct about Sahitya Akademi Award?

1. Established in 1944, It is a cultural honour that is conferred biannually by Sahitya Akademi,

India’s National Academy of letters.

2. Indian film-maker Satyajit Ray is the designer of the plaque awarded by the Sahitya Akademi.

Choose the correct answer from above

A. 1 only

B. 2 only

C. Both 1 and 2

D. Neither 1 nor 2

Answer: B

Explanation

• News - The National Academy of Letters have announced the names for the “Sahitya Akademi Award

2020 on March 12, 2021.

About

• Sahitya Akademi award established in 1954, is a literary honour that is conferred annually by Sahitya

Akademi, India’s National Academy of letters.

• Award is presented to the most outstanding books of literary merit published in any of the twenty-

four major Indian languages recognised by the Akademi (including English).

www.sleepyclasses.com

Call 6280133177Available on App store

and Play Store 15

T.me/SleepyClasses

• Sahitya Akademi award is the second highest literary honour by the Government of India, after

Jananpith award.

• The award is presented in the form of a casket containing an engraved copper- plaque, a shawl and a

cheque of Rs 1 lakh.

• Indian film-maker Satyajit Ray is the designer

of the plaque awarded by the Sahitya

Akademi.

Criteria

• The author must be of Indian Nationality.

• Book/work eligible for award must be an

outstanding contribution to the language and

literature to which it belongs.

• When equal merit for books of two or more are found, certain criterialike total literary contribution

and standing of authors shall be taken into consideration for declaring award.

12.Based on following statements identify the monument .

1. Built in the 13th century, conceived as a gigantic chariot of the Sun God, with 12 pairs of

exquisitely ornamented wheels pulled by seven horses.

2. Built by King Narasimhadeva I, the great ruler of Ganga dynasty.

3. It was included in UNESCO World Heritage Site in 1984 for its architectural greatness and also

for the sophistication and abundance of sculptural work.

Choose the correct Option

A. Jagannath Puri

B. Lingaraj Temple

C. Konark Temple

D. Lalitgiri

Answer: C

Explanation

• Built in the 13th century, the Konark temple was conceived as a gigantic chariot of the Sun God, with

12 pairs of exquisitely ornamented wheels pulled by seven horses.

• It was built by King Narasimhadeva I, the great ruler of Ganga dynasty.

• The temple was included in UNESCO World Heritage Site in 1984 for its architectural greatness and

also for the sophistication and abundance of sculptural work.

• The temple is a perfect blend of Kalinga architecture, heritage, exotic beach and salient natural

beauty.

16Available on App store

and Play Storewww.sleepyclasses.com

Call 6280133177

T.me/SleepyClasses

• It is protected under the National Framework of India by the Ancient Monuments and Archaeological

Sites and Remains

• (AMASR) Act (1958) and its Rules (1959).

• The Konark is the third link of Odisha’s Golden

Triangle.

• The first link is Jagannath Puri and the second link is

Bhubaneswar (Capital city of Odisha).

• This temple was also known as ‘BLACK PAGODA’ due

to its dark colour and used as a navigational landmark

by ancient sailors to Odisha.

13. Which of the following statements are correct about the World Heritage Sites ?

1. To be selected, a World Heritage Site must be an already classified landmark, unique in some

respect as a geographically and historically identifiable place having special cultural or physical

significance.

2. All world heritage sites in India are regulated and maintained by Ministry of Culture.

Choose the correct statements

A. 1 only

B. 2 only

C. Both 1 and 2

D. None of the above

Answer: A

Explanation

About

• A World Heritage site is classified as a natural or man-made area or a structure that is of international

importance, and a space which requires special protection.

www.sleepyclasses.com

Call 6280133177Available on App store

and Play Store 17

T.me/SleepyClasses

• These sites are officially recognised by the UN

and the United Nations Educational Scientific

and Cultural Organisation, also known as

UNESCO.

• UNESCO believes that the sites classified as

World Heritage are important for humanity, and

they hold cultural and physical significance

• The list is maintained by the international World

Heritage Programme administered by the

U N E S C O W o r l d H e r i t a g e C o m m i t t e e ,

composed of 21 UNESCO member states which

are elected by the General Assembly.

• Each World Heritage Site remains part of the legal territory of the state wherein the site is located

and UNESCO considers it in the interest of the

• To be selected, a World Heritage Site must be an already classified landmark, unique in some respect

as a geographically and historically identifiable place having special cultural or physical significance.

• CULTURAL SITES - Under Protection of Archaeological Survey of India (22).

• NATURAL SITES: (7) - Under Protection of Ministry of Environment, Forest and Climate Changes and

respectively other departments.

14.Which one of the following was the very special port in the Kaktiya dynasty ?

A. Kakinada

B. Motupalli

C. Masulipatnam

D. Nelluru

Answer: B

Explanation

• Bronze idols of Lord Nataraja, his

consort Parvati, their son Ganesha and

Alwars, the Bhakti movement saints

dating back to the 12th century, were

found in the 1970s, has remained

hitherto untapped.

• It is interesting to note that coins and

pottery from the 9th century Chinese

Ming dynasty were also found during

the excavations conducted in 1972.

18Available on App store

and Play Storewww.sleepyclasses.com

Call 6280133177

T.me/SleepyClasses

• Italian traveller Marco Polo who had visited the port in 1287 AD, had given a vivid account on the

flourishing trade activities at the port during the reign of Kakatiya queen Rudrama Devi.

• Motupalli Veerabhadraswamy temple

✓Veerabhadra Swamy temple was constructed during the regime of the Cholas,It has stone

inscriptions in Telugu and Tamil and awe- inspiring Panchaloha idols.

✓It is said that this Temple flourished during the times of Kakatiya rulers who established sea trade

with foreign countries.

15. Sanskrit prosody or Chandas is related to which of the following ?

A. Tantric Rituals

B. Architecture

C. Vedic studies

D. None of the above.

Answer: C

Explanation

About

• Sanskrit prosody or Chandas refers to one of the six Vedangas, or limbs of Vedic studies.

• It is the study of poetic metres and verse in Sanskrit.

• This field of study was central to the composition of the Vedas, the scriptural canons of Hinduism, so

central that some later Hindu and Buddhist texts refer to the Vedas as Chandas.

• The Chandas, as developed by the Vedic schools, were organized around seven major metres, and

each had its own rhythm, movements and aesthetics.

• Sanskrit metres include those based on a fixed number of syllables per verse, and those based on fixed

number of morae per verse.

• Extant ancient manuals on Chandas include Pingala's Chandah Sutra, while an example of a medieval

Sanskrit prosody manual is Kedara Bhatta's Vrittaratnakara.

16.Which of the following statements about Ayya Vaikunda Swamikal are correct ?

1. He lived in the Vijaynagara Empire .

2. He was against idol worship and he did not allow the portraitures to draw his figure.

3. He advised his followers to practice Dharma and the most important form of Dharma was

considered ‘Annadhanam’.

Choose the correct answer from above -

A. 1 and 2 only

www.sleepyclasses.com

Call 6280133177Available on App store

and Play Store 19

T.me/SleepyClasses

B. 2 and 3 only

C. 1 and 3 only

D. All of the above

Answer: B

Explanation

• Context - Prime Minister paid tributes to

Ayya Vaikunda Swamikal, a great thinker

and social reformer of the 19th century, on

his birth anniversary.

• Ayya Vaikunda (1809-1851) was a great

humanist and social thinker.

• He lived in the Princely Kingdom of

Travancore in the early decades of the 19th

Century Kerala.

• He is remembered as the first well known

social reformer in India who critiqued the

caste discrimination and religious hierarchy

and fought against the practice of untouchability.

• His gave an exhortation of 'ONE CASTE, ONE RELIGION, ONE CLAN, ONE WORLD, ONE GOD’.

• He was against idol worship and he did not allow the portraitures to draw his figure.

• His theory and practice like 'samathva samajam', 'sama panthi bhojan’, aimed at cultivating a thought

of equality and unity in Indian.

• He founded 'samathva samajam' in 1836 and was considered the first socio reform movement in

India.

• He called the rule of the British as ‘Ven Neechan’ and the rule of the king of Travancore as ‘The

Neechan of Ananthapuri’.

• He advised his followers to practice Dharma and the most important form of Dharma was considered

‘Annadhanam’.

• He organized ‘Sama Panthi Bhojana’ in each and every place of worship in the name of ‘Annadanam’.

17.Which of the following statements about Rakhigarhi are correct ?

1. The site is located in Sarasvati river plain.

2. Two levels of Early (3500 BC— 2600 BC) and Mature Harappan (2600 BC —1800 BC) civilization

have been found at Rakhigarhi.

Choose the correct answer from above -

A. 1 only

20Available on App store

and Play Storewww.sleepyclasses.com

Call 6280133177

T.me/SleepyClasses

B. 2 only

C. Both 1 and 2

D. None of the above

Answer: C

Explanation

• Context - Rakhigarhi is being developed as one of the

Five Identified m Iconic Archeological Sites.

About Rakhigarh

• The ancient site of Rakhi-Khas and Rakhi-Shahpur are

collectively known as Rakhigarhi, located on the right

bank of now dried up Palaeo- channel of Drishadvati

or Saraswati.

• Seven (07) mounds are located here.

• The site has yielded various stages of Harappan

culture and is by far one of the largest Harappan sites

in India.

• The site shows the sequential development of the

Indus culture in the now dried up Saraswati basin.

• For development of the sites and its environs, repairing of boundary wall, pathways, public amenities ,

solar Lights, Benches are being provided.

Rakhigarhi

• Rakhigarhi, in Haryana, became an

a r c h a e o l o g i c a l h o t s p o t w h e n

Amarendra Nath, former director of

the Archaeological Survey of India

(ASI), undertook excavations at the

site in 1997.

• It is the largest Indus Valley

Civilisation site in the country

• Two levels of Early (3500 BC— 2600

BC) and Mature Harappan (2600 BC

—1800 BC) civilization have been

found at Rakhigarhi. Both the phase

have yielded a rich haul of artefacts.

• It is a necropolis which has yielded

burials, important for the study of

any civilisation.

www.sleepyclasses.com

Call 6280133177Available on App store

and Play Store 21

T.me/SleepyClasses

• The site is located in the Sarasvati river plain, some 27 km from the seasonal Ghaggar river.

• In May 2012, the Global Heritage Fund, declared Rakhigarhi one of the 10 most endangered heritage

sites in Asia.

18.Based on the following statements identify the dance -

1. It is the most popular dance form of Kerala

2. The word translates as story play.

3. It is popular for its make-up and costumes.

4. The actor never speaks with his hands.

5. They usually depict the ancient stories of Mahabharata, Ramayan and other Puranas.

6. The language of the songs used for this dance is Manipravalam.

Choose the correct option -

A. Kathakali

B. Mohiniyattam

C. Kokali

D. Tirvathirakali

Answer: A

Explanation

• Context - Kathakali maestro Chemancheri Kunhiraman Nair dies at

105.

• Kathakali, is an efflorescent art form of Kerala.

• Kathakali is probably the oldest theatre forms in the world.

• It originated in the state of Kerala and the Kathakali dancers perform

a group presentation, in which they act upon various roles which are

traditionally based on themes from sacred Hindu mythology,

especially the two epics, the Ramayana and the Mahabharata.

• The actor in Kathakali never speaks expect with his hands.

• The text of the drama is sung for him and is the baseline for his

interpretation.

• The movements of the face, eyebrows and eyeballs, cheeks, the nose and the chin are minutely

worked out and various emotions are registered in a flash by each individual actor-dancer.

• The roles of women are generally played by men, though of late, women are entering the arena.

• A cylindrical drum called chenda, another horizontally held drum (maddalam), cymbals and a gong

form the musical accompaniment, with two vocalists rendering the songs.

22Available on App store

and Play Storewww.sleepyclasses.com

Call 6280133177

T.me/SleepyClasses

• The language of the songs used for Kathakali is Manipravalam.

• Though most of the songs are set in ragas based on the microtone- heavy Carnatic music, there is a

distinct style of plain-note rendition, which is known as the Sopanam style.

• This typically Kerala style of rendition takes its roots from the temple songs which used to be sung

(continues even now at several temples) at the time when Kathakali was born.

• The costume is the most distinctive characteristic of Kathakali.

• The makeup is very elaborate and the costumes are very large and heavy.

19.Which of the following statements are correct about Bamiyan Buddhas ?

1. It is also called Greco-Buddhist art, showing influences of the Hellenistic styles of classical Greek

sculptures.

2. It is situated in the high mountains of Hindkush Range in Pakistan.

3. They were called by the locals Salsal and Shamama.

Choose the correct Option -

A. 1 and 2 only

B. 2 and 3 only

C. 1 and 3 only

D. All of the above

Answer: C

Explanation

Context

• The Bamiyan Buddhas were recently brought back to life

with the help of 3D projection in an event held in the

Bamiyan Valley of Afghanistan.

• The event called 'A Night with Buddha' was held on the

20th anniversary of annihilation of the Buddha statues in

the valley.

• The event is a beautiful sight where hundreds of young

Afghan men and women, carrying kerosene lanterns, spend

their nights in a valley of central highlands.

Bamiyan Buddhas

• The Bamiyan Buddhas were great examples of a confluence of Gupta, Sassanian and Hellenistic

artistic styles.

• They are said to date back to the 5th century AD.

• They were once the tallest standing Buddhas in the world.

www.sleepyclasses.com

Call 6280133177Available on App store

and Play Store 23

T.me/SleepyClasses

• They were called by the locals as Salsal and Shamama.

• Salsal means “light shines through the universe”; Shamama is “Queen Mother”.

• UNESCO included the remains in its list of world heritage sites in 2003.

• Efforts have been made to restore and reconstruct the Buddhas in their niches with the pieces

available.

Bamiyan

• It is situated in the high mountains of the Hindu Kush in the central highlands of Afghanistan.

• The valley is set along the line of the Bamiyan River.

• It was once integral to the early days of the Silk Roads, providing passage for merchants, culture,

religion and language.

20.Toto Tribal People are found in which of the following states ?

A. Jharkhand

B. West Bengal

C. Nagaland

D. Andhra Pradesh

Answer: B

Explanation

• The Toto is an isolated tribal group residing only

in a small enclave called Totopara in the

Alipurduar district of West Bengal

• Totopara is located at the foot of the Himalayas

just to the south of the borderline between

Bhutan and West Bengal (on the western bank of

Torsa river).

• Totos were nearly becoming extinct in the 1950s,

but recent measures to safeguard their areas

from being swamped with outsiders have helped

preserve their unique heritage and also helped

the population grow.

• It is a very primitive inheritance, considered as

Mongoloid people.

• Toto People belong in the Indo- Bhutanese tribal community.

• Now a day's all Toto people concentrate in Totopara, 22 km away from Madarihat, Alipurduar District.

• They have flat nose, small eye, broad and square cheeks, thick lips and black iris.

24Available on App store

and Play Storewww.sleepyclasses.com

Call 6280133177

T.me/SleepyClasses

• Toto tribes believed in joint family system as like as other Indian communities people, but nuclear

family are also available.

• Monogamy is common form of marriage among the Toto but polygamy is not prohibited.

• They have a simple lifestyle and are largely dependent on trading vegetables and fruits.

• In spite of the fact that they define themselves as Hindus, the Totos are believers of God Ishpa and

goddess Cheima.

www.sleepyclasses.com

Call 6280133177Available on App store

and Play Store 25

T.me/SleepyClasses

3. Polity & Governance To watch the following questions on YouTube, click on the links given below

• Video 1

• Video 2

• Video 3

• Video 4

1. Which of the following is true with regard to the 69th Constitutional Amendment Act?

1. It provided a special status to the UT of Delhi and re-designatedit the National Capital Territory

of Delhi.

2. The administrator of Delhi is designated as the lieutenant(lt.) governor who appoints the Chief

Minister.

3. The assembly can make laws on all the matters of the State List and the Concurrent List except

the three matters of the State List, that is, public order, police, and land.

Select the correct answer using the codes given below

A. 1 and 2 only

B. 1 and 3 only

C. 2 and 3 only

D. All of the above

Answer: B

Explanation

• The 69th Constitutional Amendment Act of 1991 provided a special status to the Union Territory of

Delhi, and redesignated it the National Capital Territory of Delhi and designated the administrator of

Delhi as the lieutenant (lt.) governor. It created a legislative assembly and a council of ministers for

Delhi.

• The assembly can make laws on all the matters of the State List and the Concurrent List except the

three matters of the State List, that is, public order, police, and land. But, the laws of Parliament

prevail over those made by the Assembly.

• The chief minister is appointed by the President (not by the lt. governor).

• The other ministers are appointed by the president on the advice of the chief minister.

• The ministers hold office during the pleasure of the president.

• The council of ministers is collectively responsible to the assembly.

26Available on App store

and Play Storewww.sleepyclasses.com

Call 6280133177

T.me/SleepyClasses

2. The first high court was set-up in 1862 at

1. Calcutta

2. Bombay

3. Madras

4. Allahabad

Select the correct answer using the codes given below

A. 1 only

B. 1, 2 and 3 only

C. 2 and 3 only

D. All of the above

Answer: B

Explanation

• The institution of high court originated in India in 1862 when the high courts were set up at Calcutta,

Bombay, and Madras1.

• In 1866, a fourth high court was established at Allahabad.

3. What was the main issue of concern for the T.K. Vishwanathan Committee?

A. To deal with cyber crimes especially online hate speech.

B. To suggest measures to strengthen banks and reduce NPAs.

C. To deal with electoral reforms.

D. To suggest measures to improve quality of primary education.

Answer: A

Explanation

• An expert committee headed by former Lok Sabha secretary general TK Viswanathan to deal with

cyber crimes especially online hate speech recommended appointing cyber crime coordinators in all

states and establishing cyber crime cells in each district.

• The committee was formed after Supreme Court struck down Section 66 A of the Information

Technology (IT) Act, 2000.

4. Which of the following is correct about the National Green Tribunal Act, 2010?

1. The NGT is a statutory tribunal to handle the expeditious disposal of the cases pertaining to

environmental issues

2. The NGT is bound by the procedure laid down under the Code of Civil Procedure, 1908.

www.sleepyclasses.com

Call 6280133177Available on App store

and Play Store 27

T.me/SleepyClasses

3. The NGT is mandated to make and endeavour for disposal of applications or appeals finally within

6 months of filing of the same

Select the correct option

A. Only 1

B. Only 2 and 3

C. Only 1 and 3

D. 1, 2 and 3

Answer: C

Explanation

• National Green Tribunal Act, 2010 (NGT) is an Act of the Parliament of India which enables creation

of a special tribunal to handle the expeditious disposal of the cases pertaining to environmental

issues.

• The Tribunal's dedicated jurisdiction in environmental matters shall provide speedy environmental

justice and help reduce the burden of litigation in the higher courts.

• The Tribunal shall not be bound by the procedure laid down under the Code of Civil Procedure, 1908,

but shall be guided by principles of natural justice.

• The tribunal is mandated to make and endeavour for disposal of applications or appeals finally within

6 months of filing of the same.

• Initially, the NGT is proposed to be set up at five places of sittings and will follow circuit procedure for

making itself more accessible; New Delhi is the Principal Place of Sitting of the Tribunal and Bhopal,

Pune, Kolkata and Chennai shall be the other place of sitting of the Tribunal.

5. Which of the below statements are true regarding the Contingency Fund of India?

1. It is placed at the disposal of the president but requires authorisation by the Parliament.

2. It is governed by the contingency fund of the India Act in 1950.

Select the correct answer using the codes given below.

A. 1 only

B. 2 only

C. Both 1 and 2

D. Neither 1 nor 2

Answer: C

Explanation

• Contingency Fund of India (Article 267) - The Constitution authorised the Parliament to establish a

‘Contingency Fund of India’, into which amounts determined by law are paid from time to time.

28Available on App store

and Play Storewww.sleepyclasses.com

Call 6280133177

T.me/SleepyClasses

• Accordingly, the Parliament enacted the contingency fund of the India Act in 1950. This fund is placed

at the disposal of the president, and he can make advances out of it to meet unforeseen expenditures

pending its authorisation by the Parliament.

• The fund is held by the Finance secretary on behalf of the president.

6. The Opportunity Index 2021 report was recently released by

A. Transparency International

B. Google

C. LinkedIn

D. International Labour Organization

Answer: C

Explanation

Context

• It is a new report by LinkedIn.

• 9 in 10 or 89 per cent of women were negatively impacted by the coronavirus pandemic.

• About 85%, or four in five working women in India believe they have missed out on a raise, promotion,

or work offer because of their gender.This average stands at 60% for the Asia Pacific (APAC) region.

• More women in India have experienced the impact of gender on career development when compared

to the APAC region.

Challenges for women

• Lack of time and discrimination for family care.

• Gender is a barrier when it comes to attaining opportunities.

• Lack of guidance through networks.

• Workplace discrimination because of household responsibilities.

7. Which of the following statements regarding Registration of political parties are correct?

1. Registration of Political parties is governed by the provisions of Section 29A of the

Representation of the People Act, 1951 2. To be eligible for a ‘National Political Par ty of India’, it should secure at least ten percent of the

valid votes polled in any four or more states, at a general election to the House of the People or,

to the State Legislative Assembly. A. 1 only

B. 2 only

C. Both1and2 www.sleepyclasses.com

Call 6280133177Available on App store

and Play Store 29

T.me/SleepyClasses

D. Neither 1 nor 2

Answer: A

Explanation

• Context:The Election Commission has said it had reduced the public notice period for new political

parties seeking registration from 30 days to seven days due to the delays caused by the COVID-19

pandemic.

• The relaxation in notice period would remain in force till the last dates of nomination for the Assam,

Tamil Nadu, Kerala and Puducherry and West Bengal elections, that is March 19 and April 7

respectively.

• According to guidelines, the applicants are supposed to publish the proposed name of their party in

two national and local daily newspapers each on two days, seeking objections, if any, within 30 days.

Registration of political parties

• Registration of Political parties is governed by the provisions of Section 29A of the Representation of

the People Act, 1951.

• A party seeking registration under the said Section with the Election Commission has to submit an

application to the Commission within the said period following the date of its formation as per

guidelines prescribed by the Election Commission of India in exercise of the powers conferred by

Article 324 of the Commission of India and Section 29A of the Representation of the People Act,

1951.

To be eligible for a ‘National Political Party of India

• It secures at least six percent of the valid votes polled in any four or more states, at a general election

to the House of the People or, to the State Legislative Assembly.

• In addition, it wins at least four seats in the House of the People from any State or States.

• It wins at least two percent seats in the House of the People (i.e., 11 seats in the existing House having

543 members), and these members are elected from at least three different States.

To be eligible for a ‘State Political Party

• It secures at least six percent of the valid votes polled in the State at a general election, either to the

House of the People or to the Legislative Assembly of the State concerned

• In addition, it wins at least two seats in the Legislative Assembly of the State concerned.

• It wins at least three percent (3%) of the total number of seats in the Legislative Assembly of the

State, or at least three seats in the Assembly, whichever is more.

8. Which of the following statements regarding Insurance ombudsman are correct?

1. One can approach the Ombudsman with complaint only if the value of the claim including

expenses claimed is not be above Rs 30 lakhs.

30Available on App store

and Play Storewww.sleepyclasses.com

Call 6280133177

T.me/SleepyClasses

2. If a settlement by recommendation does not work, the Ombudsman will Pass an award within 3

months of receiving all the requirements from the complainant and which will be binding on the

insurance company.

A. 1 only

B. 2 only

C. Both1and2

D. Neither 1 nor 2

Answer: C

Explanation

Context:The government has amended insurance ombudsman rules.

• As per the new rules:

✓Insurance brokers are now covered under ombudsman rules.

✓Policyholders are now allowed to file online complaints.

✓The scope of complaints has been enlarged to ombudsmen from only disputes earlier to

deficiencies in service on the part of insurers, agents, brokers and other intermediaries.

✓The ombudsman may use video-conferencing for hearings.

✓The Insurance Ombudsman scheme was created by the Government of India for individual

policyholders to have their complaints settled out of the courts system in a cost-effective, efficient

and impartial way.

✓Any person who has a grievance against an insurer, may himself or through his legal heirs, nominee

or assignee, make a complaint in writing to the Insurance ombudsman.

9. Shekatkar Committee is related to which of the following:

A. ISRO “frame-up” case

B. Army pay scales

C. Restructuring railways

D. Border Infrastructure.

Answer: D

Explanation

• Context: Government has accepted and implemented three important recommendations of

Committee of Experts (CoE) under the Chairmanship of Lt General D B Shekatkar (Retd) relating to

border Infrastructure.

• Accepted recommendations:

✓Speeding up road construction,

www.sleepyclasses.com

Call 6280133177Available on App store

and Play Store 31

T.me/SleepyClasses

✓Outsourcing road construction work beyond optimal capacity of the Border Roads Organisation

(BRO).

✓Mandatory to adopt Engineering Procurement Contract (EPC) mode for execution of all works

costing more than Rs 100 crore.

✓Delegating enhanced procurement powers from Rs 7.5 crore to Rs 100 crore to BRO, for domestic

and foreign procurements.

• The land acquisition and all statutory clearances like forest and environmental clearance are also

made part of approval of Detailed Project Report (DPR).

• With the adoption of EPC mode of execution, it is mandatory to award work only when 90 per cent of

the statutory clearances have been obtained, implementing the recommendation of CoE regarding

obtaining prior clearances before the commencement of the project.

Other reforms suggested

• India’s defence budget should be in the range of 2.5 to 3 per cent of the GDP, keeping in mind possible

future threats.

• A Joint Services War College for training middle-level officers should be established.

• The Military Intelligence School at Pune be converted to a tri-service intelligence training

establishment.

10.Private property comes under which of the following?

A. Human Right

B. Legal Right

C. Fundamental Right

D. None of the above

Answer: A

Explanation

• Context:The Supreme Court recently held that a citizen’s right to own private property is a human

right and the state cannot take possession of it without following due procedure and authority of law.

Important observations made by the Court

• The state cannot trespass into the private property of a citizen and then claim ownership of the land

in the name of ‘adverse possession’.

• Grabbing private land and then claiming it as its own makes the state an encroacher.

• In a welfare state, right to property is a human right.

• A welfare state cannot be permitted to take the plea of adverse possession, which allows a trespasser

i.e. a person guilty of a tort, or even a crime, to gain legal title over such property for over 12 years.

32Available on App store

and Play Storewww.sleepyclasses.com

Call 6280133177

T.me/SleepyClasses

• The State cannot be permitted to perfect its title over the land by invoking the doctrine of adverse

possession to grab the property of its own citizens.

• The Himachal Pradesh government forcibly took over four acres of land belonging to a person at

Hamipur district to build a road in 1967.

• Even 52 years later, the state has failed to pay the compensation.

• The appellant was wholly unaware of her rights and entitlement in law, and did not file any

proceedings for compensation of the land compulsorily taken over by the state.

• When her petition was turned down by the High Court, the appellant moved the Supreme Court.

11.Which of the following statements are correct about Inner Line Permit?

1. ILP was introduced by the Bengal Eastern Frontier Regulation Act 1873 to regulate in flow of

outsiders to preserve the indigenous culture of tribes.

2. The areas falling under ILP are decided by respective state governments who then issue travel

documents to outsiders to enter these protected areas.

A. 1 Only

B. 2 only

C. Both 1 and 2

D. Neither 1 nor 2

Answer: D

Explanation

• The system of Inner Line Permit was first introduced by the Bengal Eastern Frontier Regulation Act,

1873, the British framed regulations restricting the entry and regulating the stay of outsiders in

designated areas. This was to protect the Crown’s own commercial interests by preventing “British

subjects” (Indians) from trading within these regions.

• In 1950, the Indian government replaced “British subjects” with “Citizen of India”. This was to address

local concerns about protecting the interests of the indigenous people from outsiders belonging to

other Indian states.

• The decision regarded areas under ILP is decided by central government. However, ILP letter to allow

outsiders to travel in these areas temporarily is issued by the respective state govts.

12.Which of the following is correct about the Data Governance Quality Index?

1. It assesses performance of different states of India on the implementation of Central Sector

Schemes (CS) and Centrally Sponsored Schemes (CSS).

2. It is conducted by Development Monitoring and Evaluation Office (DMEO) under NITI Aayog.

A. 1 only

www.sleepyclasses.com

Call 6280133177Available on App store

and Play Store 33

T.me/SleepyClasses

B. 2 only

C. Both 1 and 2

D. Neither 1 nor 2

Answer: B

Explanation

• DGQI survey assesses different Ministries /Departments' performance on the implementation of

Central Sector Schemes (CS) and Centrally Sponsored Schemes (CSS).

• It is conducted by Development Monitoring and Evaluation Office (DMEO) under NITI Aayog.

• Its objective is to assess data preparedness of Ministries / Departments on a standardized framework

to drive healthy competition among them and promote cooperative peer learning from best practices.

• It will immensely help improve the implementation framework of government policies, schemes and

programmes to achieve the desired goals.

13.Which of the following statements are correct about the National Commission for Protection of

Child Rights (NCPCR)?

1. The NCPCR is a statutory body which works under the aegis of Ministry of Women and Child

Development.

2. The Commission defines a child upto the age of 16 years and adolescent from 16 to 18 years of

age.

3. It tries to ensure that all laws are in consonance with Child Rights perspective as enshrined in the

Constitution of India and the UN Convention on the Rights of the Child.

A. 1 only

B. 1 and 2 only

C. 1 and 3 only

D. All of the Above

Answer: C

Explanation

• The National Commission for Protection of Child Rights (NCPCR) is an Indian statutory body

established by an Act of Parliament, the Commission for Protection of Child Rights (CPCR) Act, 2005.

The Commission works under the aegis of Ministry of Women and Child Development, GoI. The

Commission began operational on 5th March, 2007.

• The Commission is mandated under section 13 of CPCR Act, 2005 "to ensure that all Laws, Policies,

Programmes, and Administrative Mechanisms are in consonance with the Child Rights perspective as

enshrined in the Constitution of India and the UN Convention on the Rights of the Child.” As defined

by the commission, child includes person up to the age of 18 years.

34Available on App store

and Play Storewww.sleepyclasses.com

Call 6280133177

T.me/SleepyClasses

14.Consider the following statements:

1. Any persons employed with the central or state governments can’t be appointed as election

commissioners

2. Union Ministry of Home Affairs decides the election schedule for the conduct of both general

elections and bye-elections.

3. Election Commission resolves the disputes relating to splits/ mergers of recognized political

parties.

Which of the statements given above is/are correct?

A. 1 only

B. 1 and 2 only

C. 1 and 3 only

D. 3 only

Answer: C

Explanation

• Only statements 1 and 3 are correct.

• In a 2021 judgment the Supreme Court ruled ruled that any persons employed with the central or

state governments could not be appointed as election commissioners “across the length and breadth

of the country”.

• The ruling aims at ensuring independence of the election commissions. Election Commission of India

is a 3 member body and decides the election schedule for both general elections and bye-elections.

• It is also the function of election commission to resolve the disputes relating to splits/mergers of

recognized political parties.

15.In India, Judicial Review implies:

A. the power of the Judiciary to pronounce upon the constitutionality of laws and executive orders.

B. the power of the Judiciary to question the wisdom of the laws enacted by the Legislatures.

C. the power of the Judiciary to review all the legislative enactments before they are assented to by

the President.

D. the power of the Judiciary to review its own judgements given earlier in similar or different cases.

Answer: A

Explanation

• Judicial review is the power of judiciary to examine the constitutionality of legislative enactments and

executive orders of both the Central and state governments.

www.sleepyclasses.com

Call 6280133177Available on App store

and Play Store 35

T.me/SleepyClasses

• On examination, if they are found to be violative of the Constitution (ultra- vires), they can be

declared as illegal, unconstitutional and invalid (null and void). Consequently, they cannot be enforced

by the government.

• The constitutional validity of a legislative enactment or an executive order can be challenged in a high

court or supreme court on the following three grounds:

✓  it infringes the fundamental rights (Part III),

✓  it is outside the competence of the authority which has framed it, and

✓  it is repugant to the constitutional provisions

16.Consider the following activities

1. Spraying pesticides on a crop field.

2. Inspecting the craters of active volcanoes.

3. Collecting breath samples from spouting whales for DNA analysis.

At present level of technology, which of the above activities can be successfully carried out by using

drones -

Choose the correct answer from above

A. 1 and 2 only

B. 2 and 3 only

C. 1 and 3 only

D. All of the above

Answer: D

17.What is the importance of using Pneumococcal Conjugate Vaccines in India?

1. These vaccines are effective against pneumonia as well as meningitis and sepsis.

2. Dependence on antibiotics that are not effective against drug resistant bacteria can be reduced.

3. These vaccines have no side effects and cause no allergic reactions.

Choose the correct Option

A. A. 1 only

B. 1 and 2 only

C. 3 only

D. All of the above

Answer: B

36Available on App store

and Play Storewww.sleepyclasses.com

Call 6280133177

T.me/SleepyClasses

18.Which of the following are the reasons/factors for exposure to benzene pollution ?

1. Built in Automobile exhaust

2. Tobacco smoke

3. Wood burning

4. Using varnished wooden furniture.

5. Using Products made form polyurethane

Choose the correct Option -

A. 1, 2 and 3 only

B. 2 and 4 only

C. 1, 3 and 4 only

D. 1, 2, 3, 4 and 5

Answer: D

19.With reference to India’s Desert National Park, which of the following statements are correct ?

1. It is spread over two districts.

2. There is no human habitation inside the Park.

3. It is one of the natural habitats of Great Indian Bustard.

Choose the correct Option

A. 1 and 2 only

B. 2 and 3 only

C. 1 and 3 only

D. 1, 2and 3

Answer: C

20.With reference to the current trends in the cultivation of sugarcane in India, consider the following

statements -

1. A substantial saving in seed material is made when 'bud chip settlings' are raised in a nursery and

transplanted in the main field.

2. When direct planting of seeds is done, the germination percentage is better with single- budded

setts as compared to setts with many buds.

3. If bad weather conditions prevail when seeds are directly planted, single- budded seeds have

better survival as compared to large setts.

www.sleepyclasses.com

Call 6280133177Available on App store

and Play Store 37

T.me/SleepyClasses

4. Sugarcane can be cultivated using settlings prepared from tissue culture. Which of the

statements

Choose the correct Option

A. 1 and 2 only

B. 3 only

C. 1 and 4 only

D. 2, 3 and 4 only

Answer: D

38Available on App store

and Play Storewww.sleepyclasses.com

Call 6280133177

T.me/SleepyClasses

4. Economy To watch the following questions on YouTube, click on the links given below

• Video 1

• Video 2

• Video 3

• Video 4

1. Consider the following statements:

1. Capital Adequacy Ratio (CAR) is the amount that banks have to maintain in the form of their own

funds to offset any loss that banks incur if the account-holders fail to repay dues.

2. CAR is decided by each individual bank.

Which of the statements given above is/are correct?

A. 1 only

B. 2 only

C. Both of the above

D. None of the above

Answer: A

Explanation

• The Capital Adequacy Ratio (CAR) is a measure of a bank's available capital expressed as a percentage

of a bank's risk- weighted credit exposures.

• The Capital Adequacy Ratio, also known as capital-to-risk weighted assets ratio (CRAR), is used to

protect depositors and promote the stability and efficiency of financial systems around the world.

• Two types of capital are measured:

✓Tier one capital, which can absorb losses without a bank being required to cease trading, and

✓Tier two capital, which can absorb losses in the event of a winding-up and so provides a lesser

degree of protection to depositors.

• Currently, Basel III norms mandate 8% CRAR while RBI mandates minimum CRAR of 9%.

2. WTO’s Agreement on Agriculture (AoA) includes which of the following?

1. Domestic Support Commitments

2. Sanitary and Phytosanitary Measures

3. Special Safeguard Provisions

www.sleepyclasses.com

Call 6280133177Available on App store

and Play Store 39

T.me/SleepyClasses

4. Countervailing duties

Select the correct answer using the code given below:

A. 1 and 3 only

B. 1, 2 and 3 only

C. 3 only

D. All of the above

Answer: B

Explanation

• The WTO Agreement on Agriculture, which came into force in 1995, aims at reforming agricultural

trade and making it fairer and more competitive.

• The WTO Agreement on Subsidies and Countervailing Measures disciplines the use of subsidies, and

it regulates the actions countries can take to counter the effects of subsidies.

• Under the agreement, a country can use the WTO’s dispute-settlement procedure to seek the

withdrawal of the subsidy or the removal of its adverse effects. Or the country can launch its own

investigation and ultimately charge extra duty (“countervailing duty”) on subsidized imports that are

found to be hurting domestic producers.

• Its provisions and rules also relate to Special Safeguard Provisions which is a protection measure

allowed for developing countries to take contingency restrictions against agricultural imports that are

causing injuries to domestic farmers.

• Its provisions and rules also relate to Sanitary and Phytosanitary Measures and it concerns with the

application of food safety and animal and plant health regulations.

3. Which of the following are about the situation of Liquidity Trap?

1. Expansionary monetary policy does not stimulate economic growth.

2. Public would prefer to hold money in banks rather than in cash form.

3. Public would rush to buy more bonds to earn a fixed interest rate in such a situation of volatility.

Choose the correct codes given below.

A. 1 only

B. 2 and 3only

C. 1 and 2 only

D. All of the above

Answer: B

40Available on App store

and Play Storewww.sleepyclasses.com

Call 6280133177

T.me/SleepyClasses

Explanation

• A liquidity trap is a situation in which interest rates are low and savings rates are high, rendering

monetary policy ineffective.

• In a liquidity trap, consumers choose to avoid bonds and keep their funds in savings because of the

prevailing belief that interest rates will soon rise (which would push bond prices down).

• Because bonds have an inverse relationship to interest rates, many consumers do not want to hold an

asset with a price that is expected to decline.

4. Which of the following is/are the consequences that follow if there is an increase in the fixed

investment in the economy?

1. Growth in GDP

2. Decline in consumption

3. Improvement in investment outlook

4. Increase in the Interest rates

Select the correct answer using the codes given below.

A. 1 and 2 only

B. 2 and 4 only

C. 1 and 3 only

D. All of the above

Answer: C

Explanation

• The last year’s Economic Survey, describes the virtuous cycle of growth wherein an increase in the

rate of fixed investment accelerates the growth of GDP that in turn induces a higher growth in

consumption.

• Higher growth of consumption improves the investment outlook. This virtuous cycle of higher fixed

investment-higher GDP growth-higher consumption growth generates economic development in the

country.

5. Which of the following is/are incorrectly matched

1. Treasury Bills - Public Account of India

2. Kisan Vikas Patra - Consolidated Fund of India

3. Sukanya Samriddhi Account - Contingency fund of India

4. Public Provident Fund - Public Account of India

Select the correct answer using the code given below

www.sleepyclasses.com

Call 6280133177Available on App store

and Play Store 41

T.me/SleepyClasses

A. 1 only

B. 1, 2, and 3 Only

C. 2, 3 and 4 only

D. All of the above

Answer: B

Explanation

• Receipts under Public Account account mainly flow from the sale of Savings Certificates,

contributions into General Provident Fund, Public Provident Fund, Security Deposits and Earnest

Money Deposits (a kind of security deposits) received by the government.

• It also includes schemes like Kisan Vikas Patra, Sukanya Samridhi Scheme, etc.

• In respect of such deposits, the government is acting as a Banker or Trustee and refunds the money

after the completion of the contract/ event.

• All government borrowings through Treasury bills and Dated securities go to the Consolidated Fund

of India.

6. Which of the following have MSP (Minimum Selling Price)?

1. Wheat

2. Paddy

3. Sugar

4. Cotton

A. 1, 2 & 3 only

B. 2, 3 & 4 only

C. 1 & 2 only

D. All of the above

Answer: D

Explanation

• Minimum Support Price (MSP) is a form of market intervention by the Government of India to insure

agricultural producers against any sharp fall in farm prices.

The minimum support prices are announced by the Government of India at the beginning of the

sowing season for certain crops on the basis of the recommendations of the Commission for

Agricultural Costs and Prices (CACP).

• MSP is price fixed by Government of India to protect the producer - farmers - against excessive fall in

price during bumper production years.

The minimum support prices are a guarantee price for their produce from the Government.

42Available on App store

and Play Storewww.sleepyclasses.com

Call 6280133177

T.me/SleepyClasses

• The major objectives are to support the farmers from distress sales and to procure food grains for

public distribution.

In case the market price for the commodity falls below the announced minimum price due to bumper

production and glut in the market, government agencies purchase the entire quantity offered by the

farmers at the announced minimum price.

• Government announces minimum support prices (MSPs) for 22 mandated crops and fair and

remunerative price (FRP) for sugarcane. The mandated crops are 14 crops of the kharif season, 6 rabi

crops and two other commercial crops. In addition, the MSPs of toria and de-husked coconut are fixed

on the basis of the MSPs of rapeseed/mustard and copra, respectively.

• The list of crops are as follows.

✓Cereals (7) - paddy, wheat, barley, jowar, bajra, maize and ragi

✓Pulses (5) - gram, arhar/tur, moong, urad and lentil

✓Oilseeds (8) - groundnut, rapeseed/mustard, toria, soyabean, sunflower seed, sesamum, safflower

seed and nigerseed

✓Raw cotton

✓Raw jute

✓Copra

✓De-husked coconut

✓Sugarcane (Fair and remunerative price)

✓Virginia flu cured (VFC) tobacco

• To control the constant slide in ex mill prices of sugar, the central government had in June 2018

specified the concept of MSP for sugar. Back then the MSP was fixed at Rs 2,850/- per quintal which

was subsequently raised to Rs 3,100/- per quintal.

7. Which of the following statements is/are correct about Anti-dumping Duty?

1. It is imposed in order to counter the negative impact of import subsidies to protect domestic

producers.

2. A country is allowed to impose tariffs on dumped products to provide a level-playing field to

domestic manufacturers.

A. 1 only

B. 2 only

C. Both 1 and 2

D. Neither 1 nor 2

Answer: B

www.sleepyclasses.com

Call 6280133177Available on App store

and Play Store 43

T.me/SleepyClasses

Explanation

Context

• The U.S. Department of Commerce is preparing to tax aluminium sheet exporters from 18 countries

including India after determining that they had benefited from subsidies and dumping.

• The US International Trade Commission (ITC), an independent body, must approve the final decision

to impose anti-dumping or countervailing duties.

• In international trade practise, dumping happens when a country or a firm exports an item at a price

lower than the price of that product in its domestic market.

• Dumping impacts the price of that product in the importing country, hitting margins and profits of

local manufacturing firms.

• Anti-dumping duty is imposed to rectify the situation arising out of the dumping of goods and its trade

distortive effect.

• According to global trade norms, including the World Trade Organization (WTO) regime, a country is

allowed to impose tariffs on such dumped products to provide a level-playing field to domestic

manufacturers.

• Anti-dumping duty is different from countervailing duty. The latter is imposed in order to counter the

negative impact of import subsidies to protect domestic producers.

• Countervailing Duties (CVDs) are tariffs levied on imported goods to offset subsidies made to

producers of these goods in the exporting country.

• CVDs are meant to level the playing field between domestic producers of a product and foreign

producers of the same product who can afford to sell it at a lower price because of the subsidy they

receive from their government.

8. Which of the following statements are correct about Asian Infrastructure Investment Bank (AIIB)?

1. It is a multilateral development bank with a mission to improve social and economic outcomes in

Asia and beyond.

2. The Bank has established an International Advisory Panel (IAP) to coordinate with World Bank

and IMF on investment related projects worldwide and their impacts on Asian economy.

A. 1 only

B. 2 only

C. Both 1 and 2

D. Neither 1 nor 2

Answer: A

Explanation

Context

• Centre has signed $304 million pact with AIIB for power transmission network in Assam.

44Available on App store

and Play Storewww.sleepyclasses.com

Call 6280133177

T.me/SleepyClasses

• The fund will be utilised for the ‘Assam Intra- State Transmission System Enhancement Project’,

aiming to improve reliability, capacity and security of the power transmission network in the state.

• Asian Infrastructure Investment Bank (AIIB) is a multilateral development bank with a mission to

improve social and economic outcomes in Asia and beyond.

• The Parties (57 founding members) to agreement comprise the Membership of the Bank.

• It is headquartered in Beijing.

• It commenced operations in January 2016.

• By investing in sustainable infrastructure and other productive sectors today, it aims to connect

people, services and markets that over time will impact the lives of billions and build a better future.

• There are more than 100 members now

• Fourteen of the G-20 nations are AIIB members including France, Germany, Italy and the United

Kingdom.

• China is the largest shareholder with 26.61 % voting shares in the bank followed by India (7.6%),

Russia (6.01%) and Germany (4.2 %).

• The regional members hold 75% of the total voting power in the Bank.

Various organs of AIIB

• Board of Governors: The Board of Governors consists of one Governor and one Alternate Governor

appointed by each member country. Governors and Alternate Governors serve at the pleasure of the

appointing member.

• Board of Directors: Non-resident Board of Directors is responsible for the direction of the Bank’s

general operations, exercising all powers delegated to it by the Board of Governors.

• International Advisory Panel: The Bank has established an International Advisory Panel (IAP) to

support the President and Senior Management on the Bank’s strategies and policies as well as on

general operational issues.

9. Based on the following statements identify the type of curve?

1. It plots interest rates of bonds of equal credit and different maturities.

2. The three key types include normal, inverted and flat.

3. Upward sloping curve is where longer-term bonds have higher yields than short-term ones.

4. While normal curves point to economic expansion, downward sloping (inverted) curves point to

economic recession.

A. Phillips Curve

B. Rahn’s Curve

C. Wage Curve

D. Yield Curve

www.sleepyclasses.com

Call 6280133177Available on App store

and Play Store 45

T.me/SleepyClasses

Answer: D

Explanation

Context

• A yield curve is a line that plots yields

(interest rates) of bonds having equal credit

quality but differing maturity dates.

• The slope of the yield curve gives an idea of

future interest rate changes and economic

activity.

• There are three main types of yield curve

shapes: normal (upward sloping curve),

inverted (downward sloping curve) and flat.

• This yield curve is used as a benchmark for

other debt in the market, such as mortgage

rates or bank lending rates, and it is used to

predict changes in economic output and

growth.

10.Which one of the following statements appropriately describes the “fiscal stimulus”? (UPSC- 2011)

A. It is a massive investment by the Government in manufacturing sector to ensure the supply of

goods to meet the demand surge caused by rapid economic growth

B. It is an intense affirmative action of the Government to boost economic activity in the country

C. It is Government’s intensive action on financial institutions to ensure disbursement of loans to

agriculture and allied sectors to promote greater food production and contain food inflation

D. It is an extreme affirmative action by the Government to pursue its policy of financial inclusion.

Answer: B

Explanation

Context

• A ‘stimulus’ is an attempt by policymakers to kick- start a sluggish economy through a package of

measures.

• A monetary stimulus will see the central bank expanding money supply or reducing the cost of money

(interest rates), to spur consumer spending.

• A fiscal stimulus entails the Government spending more from its own coffers or slashing tax rates to

put more money in the hands of consumers.

• With monetary policy, both conventional and unconventional, having reached the limits of its

effectiveness in most of the advanced industrial countries, the only instrument left for boosting

demand is fiscal policy.

46Available on App store

and Play Storewww.sleepyclasses.com

Call 6280133177

T.me/SleepyClasses

• There are calls for a government stimulus package to revitalize the economy.

11.In context of Flexible Inflation Targeting Framework, which of the following is/are correct:

1. India has a flexible inflation targeting framework which came into after the 2016 amendment to

the Reserve Bank of India (RBI) Act, 1934.

2. The amended RBI Act provides for the inflation target to be set by the Government of India, in

consultation with the Reserve Bank, once every five years.

A. 1 only

B. 2 only

C. Both are correct

D. None of the above

Answer: C

Explanation

• Flexible Inflation Targeting Framework:

• Now there is a flexible inflation targeting framework in India (after the 2016 amendment to the

Reserve Bank of India (RBI) Act, 1934).

• The amended RBI Act provides for the inflation target to be set by the Government of India, in

consultation with the Reserve Bank, once every five years.

• The Central Government has notified 4 per cent Consumer Price Index (CPI) inflation as the target for

the period from August 5, 2016, to March 31, 2021, with the upper tolerance limit of 6 per cent and

the lower tolerance limit of 2 per cent.

• The Reserve Bank of India, in the Report on Currency and Finance for FY21, has said the current

inflation target of 4% with a +/-2% tolerance band is appropriate for the next five years.

12.Which of the following is/are correct about Vivad Se Vishwas scheme:

1. It is an indirect tax dispute resolution scheme.

2. It was enacted primarily with the objective to reduce pending GST litigations that arose after

various indirect taxes were subsumed in the GST.

A. 1 only

B. 2 only

C. Both are correct

D. None is correct

Answer: D

www.sleepyclasses.com

Call 6280133177Available on App store

and Play Store 47

T.me/SleepyClasses

Explanation

• The Income Tax Department extended the deadline for filing declarations and making payment under

the direct tax dispute resolution scheme ‘Vivad Se Vishwas’ (VsV) till March 31 and April 30.

• The Direct Tax ‘Vivad se Vishwas’ Act, 2020 was enacted on March 17, 2020, with the objective to

reduce pending income tax litigation, generate timely revenue for the government and to benefit

taxpayers.

• The scheme aims to end litigation and legacy disputes under the direct taxes category as ₹9.32 lakh

crore worth of revenue is blocked in approximately 4.8 lakh appeals pending at various income tax

appellate forums.

• The entities who opt for the scheme have to pay a requisite tax following which all litigation against

them are closed by the tax department and penal proceedings dropped.

• The response to the scheme gains significance given earlier schemes like Kar Vivad Samadhan

Scheme, 1998 (KVSS) and Direct Tax Dispute Resolution Scheme, 2016 (DTDRS) did not yield much

results.

• The scheme has reportedly received an overwhelming response with a settlement amount of over Rs

97,000 crore as of February.

• So far, over 1,25,144 of the 5,10,491 long-pending cases have been settled under the scheme.

• These comprise 24.5 per cent of the total number of such cases which were pending before the

scheme was introduced.

13.Which of the following is/are correct in context of Asia Economic Dialogue?

1. Asia Economic Dialogue (AED) is the flagship geo-economics conference of the Indian Ministry of

Finance and RBI.

2. The dialogue focuses on trade and finance dynamics in Asia and its extended neighbourhood.

A. 1 only

B. 2 only

C. Both are correct

D. None is correct

Answer: B

Explanation

• The 5th edition of the Asia Economic Dialogue (AED) was held virtually.

• About Asia Economic Dialogue (AED)

• Asia Economic Dialogue (AED) is the flagship geo- economics conference of the Indian Ministry of

External Affairs (MEA).

• The dialogue focuses on trade and finance dynamics in Asia and its extended neighbourhood.

48Available on App store

and Play Storewww.sleepyclasses.com

Call 6280133177

T.me/SleepyClasses

• It is jointly organized by the Ministry of External Affairs (MEA) and Pune International Centre (PIC).

• Theme: “Post Covid-19 Global Trade and Finance Dynamics”.

• The dialogue focuses on the pandemic’s impact on trade and finance, on the world, and on Asia in

particular.

• It also discusses strategies to deal with this impact.

14.Which of the following statements regarding Bad Bank is/are correct?

1. A bad bank is a bank set up to buy the bad loans and other illiquid holdings of another financial

institution.

2. The entity holding significant nonperforming assets will sell these holdings to the bad bank at

market price.

3. By transferring such assets to the bad bank, the original institution may clear its balance sheet

without taking write-downs.

A. 1 and 2 only

B. 2 and 3 only

C. 1 and 3 only

D. 1 ,2 and 3

Answer: A

Explanation

• A bad bank is a bank set up to buy the bad loans and other illiquid holdings of another financial

institution.

• The entity holding significant nonperforming assets will sell these holdings to the bad bank at market

price.

• By transferring such assets to the bad bank, the original institution may clear its balance sheet—

although it will still be forced to take write-downs.

• Indian banks’ pile of bad loans is a huge drag on the economy.

• It’s a drain on banks’ profits. Because profits are eroded, public sector banks (PSBs), where the bulk of

the bad loans reside, cannot raise enough capital to fund credit growth.

• Lack of credit growth, in turn, comes in the way of the economy’s return to an 8% growth trajectory.

Therefore, the bad loan problem requires effective resolution.

• The Indian Banks’ Association (IBA) has begun identifying bad loans which can be transferred to the

Centre’s proposed bad bank.

• The IBA has written to banks asking them for a list of all bad loans worth Rs 500 crore and above to

“identify magnitude of the problem” and “get clarity over initial capital required for the entity”.

www.sleepyclasses.com

Call 6280133177Available on App store

and Play Store 49

T.me/SleepyClasses

• Finance Minister had proposed setting up of a bad bank during her Union Budget 2021 speech on

February 1.

• She said the proposed entity would take over stressed loans from banks to sell to alternative

investment funds (AIF).

15.Which of the following statements are correct about Fugitive economic offender?

1. A person can be named an offender under the law if there is an arrest warrant against him or her

for involvement in economic offences involving at least Rs. 1000 crore or more and has fled from

India to escape legal action.

2. The investigating agencies have to file an application in a Special Court under the Prevention of

Money-Laundering Act containing details of the properties to be confiscated, and any

information about the person’s whereabouts.

A. 1 only

B. 2 only

C. Both 1 and 2

D. Neither 1 nor 2

Answer: A

Explanation

Fugitive economic offender

• A person can be named an offender under the law if there is an arrest warrant against him or her for

involvement in economic offences involving at least Rs. 100 crore or more and has fled from India to

escape legal action.

• The investigating agencies have to file an application in a Special Court under the Prevention of

Money-Laundering Act containing details of the properties to be confiscated, and any information

about the person’s whereabouts.

• The Special Court will issue a notice for the person to appear at a specified place and date at least six

weeks from the issue of notice.

• Proceedings will be terminated if the person appears.

• If not the person would be declared as a Fugitive Economic Offender based on the evidence filed by

the investigating agencies.

• The person who is declared as a Fugitive Economic Offender can challenge the proclamation in the

High Court within 30 days of such declaration according to the Fugitive Economic Offenders Act,

2018.

50Available on App store

and Play Storewww.sleepyclasses.com

Call 6280133177

T.me/SleepyClasses

16.Which of the following statements about “Volcano Bonds” are incorrect?

1. They are issued in order to reduce volatility in market when the economy is experiencing

stagflation.

2. It is a type of Insurance Linked Subsidy.

A. 1 only

B. 2 only

C. Both are correct

D. None of the above is incorrect

Answer: A

Explanation

• The Danish Red Cross announced that it has launched first of its kind catastrophe bond for the

volcano-related disasters in collaboration with the several financial firms.

• Volcano Bonds will enable the disaster relief agency to get the financial aid quickly to those who are

suffering because of the eruption of 10 volcanoes namely Chile, Ecuador, Cameroon, Colombia,

Mexico, Guatemala and Indonesia.

• The project’s partners aim to raise $3 million with the launch of this bond.

• This money will be transferred to the Denmark’s branch of the Red Cross.

• The Initial investors in the bond are Plenum investments and Schroder Investment Management.

• The firm specializes in the insurance-linked securities.

• The pay-out of the bond is based on the quantitative model which predict about where the funds will

be needed on the basis of the height of the ash cloud after the volcanic eruption and prevailing winds.

• The project has been in the making since the year November 2018 when it was started around a table

in Zurich. The catastrophe bonds insure against the damage from natural disasters such as

earthquakes and storms. Some of the bonds also include the volcanic eruptions in baskets of the

covered perils.

• These bonds offer a high yield to the investors who are going through the risk of losing some or all of

their investment during the disaster.

• These pandemic bonds, which were sold in the year 2017, started being criticised when they failed to

pay out during the Ebola outbreak in 2019 and in the initial phases of coronavirus pandemic in 2020.

• These bonds also went into losses amid the Covid-19 crisis.

17.The economic system aimed at eliminating waste and the continual use of resources by employing

reuse, sharing, repair, refurbishment, remanufacturing and recycling to create a closed-loop system,

minimising the use of resource inputs and the creation of waste, pollution and carbon emissions is

known as

A. Green Economy www.sleepyclasses.com

Call 6280133177Available on App store

and Play Store 51

T.me/SleepyClasses

B. Sustainable Economy

C. Circular Economy

D. Linear Economy

Answer: C

Explanation

• The government has set up 11 committees which will be led by the concerned line ministries apart

from officials from Ministry of Environment, Forest and Climate Change (MoEFCC) and NITI Aayog.

The committee will expedite the transition India from a linear to a circular economy across the 11

focus areas.

• These committees will be preparing the comprehensive action plans to help in transitioning from the

linear to the circular economy in the respective focus areas.

• The committee will also carry the necessary modalities which in turn will ensure the effective

implementation of findings and recommendations.

• The government has selected 11 focus area to move from linear economy to the circular economy.

• The areas include: Municipal Solid Waste & Liquid Waste, Electronic Waste, Scrap Metal (Ferrous &

Non- Ferrous), Solar Panels, Gypsum, Lithium-Ion Batteries, Agriculture Waste, Toxic Hazardous

Industrial Waste, Used Oil Waste, Tyre

and Rubber Recycling and End-of-life

Vehicles (ELVs).

• Sustainable growth is the key element of

the Aatma-Nirbhar Bharat initiative.

• So, India needs a development model for

the optimum utilization of resources.

Thus, in the light of climate change,

growing population, rapid urbanization,

and environmental pollution, there is a

need to move towards the circular

economy.

• Circular economy or circularity is an

e c o n o m i c s y s t e m w h i c h a i m s t o

eliminating the waste and continual use

of resources.

• This system accounts for reuse, repair,

r e f u r b i s h m e n t , r e m a n u f a c t u r i n g ,

recycling and sharing in order to create a closed-loop system.

• Thus, it minimises the use of resource inputs. It also minimises the creation of pollution, waste and

carbon emissions.

• If India adopts the circular economy path, it could bring in the substantial annual benefits. It will help

in reducing the congestion and pollution.

52Available on App store

and Play Storewww.sleepyclasses.com

Call 6280133177

T.me/SleepyClasses

18.Which of the following statements about At-1 bonds are correct?

1. AT1 bonds are the hybrid products which offer a fixed return which can be reset.

2. The bonds do not have fixed maturity.

3. These bonds have high liquidity because of which they are sold in market at high profits.

A. 1 and 2 only

B. 1 only

C. 2 only

D. 1, 2 and 3 only

Answer: A

Explanation

• Department of Financial Services (DFS), under finance ministry, has sent a memorandum to Securities

and Exchange Board of India (Sebi) and has asked it to withdraw a rule treating AT1 bonds

(perpetuals) which is having 100-year maturity.

• The circular was issued by SEBI regarding the AT1 bonds and this will take effect from April 1, 2021.

• This circulation has generated significant apprehension in mutual fund industry for which the losses

would result from the consequential revaluation of AT1 bonds.

• AT1 bonds are the hybrid products which offer a fixed return which can be reset. These bonds are

however risky as equities.

• These are unsecured instruments and are complex in nature. The investors are required to read the

fine print before they put money in them.

• The bonds do not have fixed maturity.

• However, the banks which are issuing them can repay them at certain dates.

• These specified dates are regarded as the maturity dates historically.

• The AT1 bonds with the 100 years of maturity was being regarded as a risk.

• A change in the maturity to 100 years had the potential to increases the interest rate sensitivity of the

portfolio of the fund.

• This, the investors were vulnerable to losses in case of even a small increase in interest rates.

• Thus, the mutual funds have expressed fears of increase in redemptions by investors because of fear

of such losses.

• On the other side, the bonds are of relatively low liquidity because of which it is hard to sell them.

19.Which of the following statements are incorrect about Participatory Notes?

1. The participatory notes are also regarded as offshore derivative instruments as they are used by

the investors abroad but not within India.

www.sleepyclasses.com

Call 6280133177Available on App store

and Play Store 53

T.me/SleepyClasses

2. Participatory notes are non- transferable.

A. 1 only

B. 2 only

C. Both 1 and 2

D. Neither 1 nor 2

Answer: B

Explanation

• Participatory Notes which are also regarded as P-Notes (PNs) are the instruments issued by

registered FIIs. The PNs are issued for the overseas investors who want to invest in the stock markets

in India, without being registered under SEBI.

• In the year 2007, the percentage of participatory notes in FIIs were almost 50%.

• The participatory notes are also regarded as offshore derivative instruments as they are used by the

investors abroad but not within India.

• They are used by the clients of Foreign Institutional Investors (FIIs) who do not wish to participate

directly in the Indian stock market.

• Foreign Institutional investors (FIIs) are the entities established outside India that are responsible for

making investment proposals in India.

• They play an important role in the economy of a country.

• During 1996-97, the following changes were made in the SEBI Regulations, 1995 to facilitate the

inflow of foreign portfolio investment:

• Each of the Foreign Institutional investors can now invest up to 10% of the equity of any one

company, subject to the overall limit of 24% on investments by all FIIs, NRIs and OCBs.

• The FIIs have been permitted to invest 100% of their portfolios in debt securities under the approval

of SEBI.

• The FIIs that are eligible under SEBI are permitted to include the endowments, university funds,

foundations, charitable trusts and societies registered with a statutory authority of their country and

having a track record of 5 years.

Advantages of participatory notes

• The participatory notes play an important role in the Indian Economy.

• About 45 % of the total investments are made through the participatory notes by the foreign

instructional investors (FIIs).

• P-Notes also helps in keeping the investor’s name anonymous along with reducing the transaction

costs.

• Some of the major advantages of participatory notes are as follows:

54Available on App store

and Play Storewww.sleepyclasses.com

Call 6280133177

T.me/SleepyClasses

• Any entity can invest in the participatory notes without registering under SEBI while registering

under SEBI is compulsory for all FIIs. PNs also enable the large hedge funds to continue their

operations without disclosing their identity.

• Participatory notes are transferable through endorsement and delivery making trading easy in the

country.

• Some of the entities route their investment through participatory notes to take advantage of the tax

laws of certain preferred countries.

20.Which of the following statements about Enemy properties act are incorrect?

1. The Tashkent Declaration of January 10, 1966 included a clause that said India and Pakistan

would discuss the return of the property and assets taken over by either side in connection with

the conflict

2. Under the Defence of India Rules framed under The Defence of India Act, 1962, the Defence

ministry notifies the enemy property rules allowing custodians to detect, preserve and control of

such wealth.

A. 1 only

B. 2 only

C. Both 1 and 2

D. Neither 1 nor 2

Answer: B

Explanation

• The Center has identified over 3,000 new enemy properties left behind by the Chinese and Pakistani

nationals after the India-China war in 1962 and Indo-Pak wars in 1965 and 1971.

• The Home Ministry has notified the enemy property rules under the Enemy Property Act, 1968,

allowing custodians to detect, preserve and control of such wealth.

• In the wake of the Chinese aggression in 1962, CEPI was called upon to take charge of the Chinese

assets, with the aim of attaching the properties under the Defence of India Rules, 1962.

• In 1968, the Enemy Property Act was enacted to deal with such properties. The Act was further

amended in 1977 and 2017.

• Uttar Pradesh tops the list of Pakistani enemy properties .

• West Bengal has the second-highest number of immovable enemy properties left behind by either

Pakistani or Chinese nationals.

• Besides immovable properties, enemy properties also consist of shares, gold and silver ornaments.

• Under the Defence of India Rules framed under The Defence of India Act, 1962, the Government of

India took over the properties and companies of those who took Pakistani nationality.

www.sleepyclasses.com

Call 6280133177Available on App store

and Play Store 55

T.me/SleepyClasses

• Under the Defence of India Rules framed under The Defence of India Act, 1962, the Government of

India took over the properties and companies of those who took Pakistani nationality.

• These “enemy properties” were vested by the central government in the Custodian of Enemy

Property for India. The same was done for property left behind by those who went to China after the

1962 Sino-Indian war.

• The Tashkent Declaration of January 10, 1966 included a clause that said India and Pakistan would

discuss the return of the property and assets taken over by either side in connection with the conflict.

• However, the Government of Pakistan disposed of all such properties in their country in the year

1971 itself.

• The Enemy Property Act, enacted in 1968, provided for the continuous vesting of enemy property in

the Custodian of Enemy Property for India. Some movable properties too, are categorised as enemy

properties.

• The 2017 amended Act expanded the definition of the term “enemy subject”, and “enemy firm” to

include the legal heir and successor of an enemy, whether a citizen of India or a citizen of a country

which is not an enemy; and the succeeding firm of an enemy firm, irrespective of the nationality of its

members or partners.

• The amended law provided that enemy property shall continue to vest in the Custodian even if the

enemy or enemy subject or enemy firm ceases to be an enemy due to death, extinction, winding up of

business or change of nationality, or that the legal heir or successor is a citizen of India or a citizen of a

country which is not an enemy.

• The Custodian, with prior approval of the central government, may dispose of enemy properties

vested in him in accordance with the provisions of the Act, and the government may issue directions

to the Custodian for this purpose.

56Available on App store

and Play Storewww.sleepyclasses.com

Call 6280133177

T.me/SleepyClasses

5. Environment To watch the following questions on YouTube, click on the links given below

• Video 1

• Video 2

• Video 3

• Video 4

1. Which of the following is/are correctly matched?

1. Blue Hydrogen - Renewable Energy

2. Gray Hydrogen - Coal Gasification

3. Brown Hydrogen - Natural Gas

Choose the correct option from below?

A. 1 only

B. 2 and 3 only

C. All of the above

D. None of the above

Answer: D

Explanation

• Hydrogen, in itself, is a clean fuel.

• Manufacturing hydrogen fuel, however, is energy-intensive and has carbon byproducts.

• What is now called brown hydrogen is created through coal gasification. At the moment, it’s mainly

produced industrially from natural gas, which

• generates significant carbon emissions. That type is known as “grey” hydrogen.

• A cleaner version is “blue” hydrogen, for which the carbon emissions are captured and stored, or

reused.

• The cleanest one of all is “green” hydrogen, which is generated by renewable energy sources without

producing carbon emissions in the first place.

2. Consider he following statements related to Simplipal National Park

1. It derives its name from ‘Simul’ , a silk cotton tree.

2. It was declared a biosphere reserve but is not a part of the UNESCO World Network of Biosphere

Reserve.

www.sleepyclasses.com

Call 6280133177Available on App store

and Play Store 57

T.me/SleepyClasses

Which of the above is/are correct?

A. 1 only

B. 2 only

C. Both 1 and 2

D. Neither 1 nor 2

Answer: A

Explanation

• Similipal, which derives its name from ‘Simul’ (silk cotton) tree, is a national park and a tiger reserve

situated in the northern part of Odisha’s Mayurbhanj district.

• Similipal and the adjoining areas, comprising 5,569 sq km, was declared a biosphere reserve by the

Government of India on June 22, 1994, and lies in the eastern end of the eastern ghat.

• It was declared a biosphere reserve by the Government of India in June, 1994 and has been part of

the UNESCO World Network of Biosphere Reserve since 2009.

3. Consider the following statements regarding TX2 award

1. Pilibhit Tiger Reserve recently bagged TX2 award.

2. It has been awarded for having doubled the number of tigers in less than the stipulated time.

Which of the above is/are correct?

A. 1 only

B. 2 only

C. Both 1 and 2

D. Neither 1 nor 2

Answer: C

Explanation

• Pilibhit Tiger Reserve in UP, has won the inaugural TX2 Award, presented by CATS, Global Tiger

Forum, IUCN, UNDP, The Lion’s Share and WWF, for its remarkable contribution to tiger

conservation.

• The awards celebrate the 10-year anniversary of all 13 Tiger Range Countries committing to double

the global population of wild tigers by 2022 - a goal known as TX2.

• The TX2 goal is one of the most ambitious conservation goals ever set for a single species and the

Tiger Summit in 2022, the next Year of the Tiger, offers the opportunity to set a new vision to secure

their future.

58Available on App store

and Play Storewww.sleepyclasses.com

Call 6280133177

T.me/SleepyClasses

4. To increase the use of Solar Energy as a dependable source of electricity in India, it is important that

there be innovation in the field of batteries and storage. One such innovation in news is of Salt

Water Batteries. In their regard, which of the following is/are correct?

1. Though these batteries employ cost-effective salts, but these salts are highly toxic in nature.

2. These salt water batteries have lower energy storing density than conventional Lithium Ion

batteries.

Choose the correct option from below

A. 1 only

B. 2 only

C. Both are correct

D. None of the above

Answer: B

Explanation

• The issue of battery storage needs focus for harvesting solar energy.

Unlike conventional energy, renewable energy (such as solar) cannot be generated with precision,

since

• the sun is out only for a few hours in a day. It needs to be stored so it can be used when needed.

• It shall be done without significantly escalating the cost of the energy.

• India’s cumulative battery requirements between 2026 and 2030 will be at least 2,410 GWh.

• While Lithium-ion batteries are lighter and more compact than lead acid batteries, newer tech such as

saltwater batteries are being increasingly considered.

Salt Water Batteries

• These batteries don’t contain heavy metals, relying instead on normal saltwater electrolytes.

• While the former need to be disposed of with special processes, a saltwater battery can be easily

recycled.

• Comparatively, Lithium-ion-batteries have a relatively high capacity, which means they hold a large

amount of charge, but do not discharge or recharge their energy quickly.

• They also contain organic electrolytes and other materials that can be hazardous and flammable,

meaning they require careful handling and disposal.

5. Consider the following statements regarding UN HABITAT

1. It is the United Nations agency for human settlements and sustainable urban development.

2. It reports to the United Nations General Assembly.

Which of the above is/are incorrect?

www.sleepyclasses.com

Call 6280133177Available on App store

and Play Store 59

T.me/SleepyClasses

A. 1 only

B. 2 only

C. Both 1 and 2

D. Neither 1 nor 2

Answer: D

Explanation

• The United Nations Human Settlements Programme (UN–Habitat) is the United Nations agency for

human settlements and sustainable urban development.

• It was established in 1978 as an outcome of the First UN Conference on Human Settlements and

Sustainable Urban Development (Habitat I) held in Vancouver, Canada in 1976.

• It is mandated by the United Nations General Assembly to promote socially and environmentally

sustainable towns and cities with the goal of providing adequate shelter for all.

• It is a member of the United Nations Development Group.

• UN-Habitat maintains its headquarters at the United Nations Office at Nairobi, Kenya.

• The twin goals of the Habitat Agenda are adequate shelter for all and the development of sustainable

human settlements in an urbanizing world.

• UN-Habitat reports to the United Nations General Assembly.

6. Based on the following statements identify the species

1. Also known as Pterocarpus santalinus is an endemic tree of South India

2. They are found in Tropical Dry Deciduous forest of the Palakonda and Seshachalam hill ranges of

Andhra Pradesh and also found in Tamil Nadu and Karnataka.

3. It usually grows in the rocky, degraded and fallow lands with Red Soil and hot and dry climate.

4. IUCN has put it under the category of ‘near threatened’ species in the Red List due to the

dwindling population because of illegal felling and smuggling.

5. Its export is banned in India in accordance with the CITES and Wildlife Protection Act 1972.

6. It is used for various purposes such as immunity medicine, furniture, radiation absorbent, musical

instrument, food dyes and spices, Ayurveda and Siddha medicine, decorative and ornamental

purposes etc.

A. Amboyna wood

B. Red Sanders

C. Malabar kino

D. Burma padauk

Answer: B

60Available on App store

and Play Storewww.sleepyclasses.com

Call 6280133177

T.me/SleepyClasses

Explanation

• The Puttur sub-division police and Red Sanders Task Force personnel, in a joint operation, seized 2.5

tonnes of red sanders logs, worth ₹3 crore in the international market, at a forest location in KVB

Puram mandal.

• Red Sanders Anti Smuggling Task Force

✓Andhra Pradesh Red Sanders Anti-Smuggling Task Force was set up in November 2014 with its

base in Tirupati.

✓It claims to have curbed wood felling and smuggling to some extent in the last two years in Chittoor

and Nellore districts.

• Red Sanders (Red sandalwood or Saunders wood)

✓The scientific name for Red Sanders is Pterocarpus santalinus.

✓It is a tree species endemic to South India (hot and dry climate).

✓It is valued for its colour.

✓It is medicinally, scientifically and ornamentally very important and essential tree.

✓It grows in rocky, degraded and fallow land with Red soil.

• Conservation status

✓It is listed as an Near Threatened by the

✓IUCN because of overexploitation of its timber in South India.

✓It is also listed in the appendix II of the CITES which means that a certificate is required in order to

export it.

✓Certificate is granted only if the trade is not detrimental to the survival of the species.

7. Which of the following statements about CITES are correct ?

1. The Convention on International Trade in Endangered Species of Wild Fauna and Flora (CITES) is

an international regulatory treaty where wild species are categorised in Appendices I to III.

2. It is also known as Washington Convention.

Which of the above statements is/are correct?

A. 1 only

B. B. 2 only

C. Both of the above

D. None of the above

Answer: C

Explanation

www.sleepyclasses.com

Call 6280133177Available on App store

and Play Store 61

T.me/SleepyClasses

• The world is observing and celebrating the World Wildlife Day 2021 on March 3, 2021.

• The day is being celebrated under the theme - Forests and Livelihoods: Sustaining People and Planet.

• This theme was decided by United Nations.

• The day is celebrated in order to raise awareness with respect to the issue around the wildlife.

• The day seeks to make people realize the importance of forests and ecosystems.

• The United Nations General Assembly (UNGA) has passed a resolution at its 68th session on

December 20, 2013.

• In its resolution UN 68/205, the UNGA decided to proclaim the March 3 as World Wildlife Day.

• This proclamation was suggested by Thailand.

• March 3 was chosen because the day marks the “adoption of Convention on International Trade in

Endangered Species of Wild Fauna and Flora (CITES)”.

• The CITES was adopted on March 3, 1972 to raise awareness and benefit the fauna and flora.

• This convention is also known as Washington Convention.

• It is a multilateral treaty that seeks to protect the endangered plants and animals.

• The convention was drafted after the adoption of a resolution in 1963 during a meeting of members

of the International Union for Conservation of Nature (IUCN).

• The convention came into force in 1975.

• The convention works primarily through a system of

• classification and licensing.

• Wild species are categorised in Appendices I to III.

• This often reflects species’ threat status on the Red List of the IUCN, the International Union for

Conservation of Nature’s Red List of Threatened Species first created in 1964.

• Appendix I prohibits trade in species classified as highly endangered.

• Appendix II allows trade under very specific conditions. This requires exporting countries obtain a

permit, but not the importing country.

• Appendix III species require only a certificate of origin to be traded. National CITES management

authorities may issue permits once scientific authorities show non-detriment findings.

• CITES is legally binding on state parties to the convention, which are obliged to adopt their own

domestic legislation to implement its goals.

8. Which of the following statements about species recovery programme are correct ?

1. It is one of the components of the Integrated Development of Wildlife Habitats (IDWH).

2. It is meant for providing support to the species that are enlisted in the Red data book.

A. 1 only

62Available on App store

and Play Storewww.sleepyclasses.com

Call 6280133177

T.me/SleepyClasses

B. 2 only

C. Both of the above D.

D. None of the above

Answer: A

Explanation

• The National Board for Wildlife and Union Ministry of Environment, Forest and Climate Change last

recently included the caracal, a medium-sized wildcat found in parts of Rajasthan and Gujarat, in the

list of critically endangered species.

• The recovery programme for critically endangered species in India now includes 22 wildlife species.

Caracal

• Besides India, the caracal is found in several dozen countries across Africa, the Middle East, Central

and South Asia.

• While it flourishes in parts of Africa, its numbers in Asia are declining.

• The wildcat has long legs, a short face, long canine teeth, and distinctive ears — long and pointy, with

tufts of black hair at their tips.

• The iconic ears are what give the animal its name — caracal comes from the Turkish karakulak,

meaning ‘black ears’.

• In India, it is called siya gosh, a Persian name that translates as ‘black Ear’.

• It finds mention in Abul Fazl’s Akbarnama, as a hunting animal in the time of Akbar (1556-1605).

• Descriptions and illustrations of the caracal can be found in medieval texts such as the Anvar- i-

Suhayli, Tutinama, Khamsa-e-Nizami, and Shahnameh.

Species Recovery Programme

• It is one of the three components of the

Integrated Development of Wildlife Habitats

(IDWH).

• IDWH was started in 2008-09 as a Centrally

sponsored Scheme.

• It is meant for providing support to protected

areas (national parks, wildlife sanctuaries,

conservation reserves and community reserves

except tiger reserves), protection of wildlife

outside protected areas and recovery programmes for saving critically endangered species and

habitats.

www.sleepyclasses.com

Call 6280133177Available on App store

and Play Store 63

T.me/SleepyClasses

9. Filter feeder shark is also known as

A. Blue Whale

B. Basking Shark

C. Greenland Whale

D. Whale Shark

Answer: D

Explanation

• Fishers in Odisha rescued a whale shark for a second time in a week in Odisha. The Odisha coast is

very important for marine megafauna like turtles, whales and sharks.

• The dotted whale shark is the largest fish in the ocean and often called ‘Gentle Giant’ in fishing

parlance.

• Whale sharks are a protected species under the Wild Life (Protection) Act, 1972 and also part of the

International Union for Conservation of Nature’s Red List.

• The whale shark is a ‘filter feeder shark’ which means it does not eat meat like other sharks.

• Whale sharks filter sea water and feed on tiny planktons.

• The largest whale shark aggregation in India is along the Gujarat coast.

• On the Bay of Bengal coast, the only whale shark aggregation is in Andhra Pradesh.

• Accidental entanglement in fishing nets is a major threat to this animal.

• Whale sharks have no commercial importance. However, fishers illegally extract their fins and livers.

• Gujarat Forest Department started a compensation programme in 2005, paying Rs 25,000 for every

net damaged by a whale shark.

10.Food Waste Index Report-2021 was recently released by

A. A. Food and Agriculture Organization

B. United Nations Environment Programme

C. The World Bank Group

D. International Fund for Agricultural Development

Answer: B

Explanation

• The United Nation Environment Programme (UNEP) published the “Food Waste Index Report, 2021”.

• As per the UNEP report, estimated 931 million tonnes of food were wasted across the world 2019.

• The agency also highlighted that this amount of wasted food can circle the Earth seven times.

• The index report was published by the UNEP in collaboration with the partner organisation WRAP.

64Available on App store

and Play Storewww.sleepyclasses.com

Call 6280133177

T.me/SleepyClasses

• Out of the total waste generation, households accounts for the sixty-one per cent of waste, food

services accounts for 26 per cent from foo waste while, the retail accounts for 13 per cent waste.

• The report also highlights 17 per cent of total global food production is wasted.

• The agency further highlighted that, on the global per capita stage 121 kgs of consumer food is

wasted per year out of which 74 kgs waste is generated from the households.

11.Based on the following statements identify the river?

1. It flows through the city of Jammu.

2. It is a major left bank tributary of the river Chenab.

3. It originates from the lap of Kailash Kund glacier and adjoining area southwest of Bhadarwah in

Doda district.

4. It is a belief of the Hindus of Jammu city that the river was brought to Jammu by 'Raja Pehar

Devta' to cure his father and was given the throne of Jammu City and was declared as 'Raja', i.e.

King of Jammu with the blessings of 'Bawe wali Shri Mata Kali Ji'.

A. Marudasar

B. Tawi

C. Bhaga

D. Chandra

Answer: B

Explanation

• The Friends of River Tawi (FORT) Movement, organized a mega clean-Up drive, by the name of “Tawi

Cleanathon” as a part of their 5th week celebrations.

• After traversing Jammu city, the river crosses into Pakistan's Punjab and joins Chenab river.

• Tawi is a major left bank tributary of river Chenab.

• Tawi river transverses through and divides Jammu city in two parts -

✓The old Jammu town is located on the hill overlooking river Tawi.

✓The new town is across the river.

• Tawi river is a major source of drinking water for the old city.

• Untreated sewage in Jammu pollutes Tawi river as it passes through the city.

• Jammu and Kashmir government is in the process of constructing an artificial lake on River Tawi at

Bhagwati Nagar in Jammu for promotion of tourism and providing water supply to dry areas of the

city.

• A team of Pakistani officials visited the site and studied the details of the project which will help allay

any apprehensions about violation of provisions of Indus Water Treaty .

www.sleepyclasses.com

Call 6280133177Available on App store

and Play Store 65

T.me/SleepyClasses

12.Which of the following statements are correct about Simlipal Protected Reserve?

1. Simlipal derives its name from ‘Simul’ (silk cotton) tree.

2. It is a part of the Eastern ghats.

3. This protected area is part of the UNESCO World Network of Biosphere Reserves

4. It includes three protected areas — Simlipal Tiger Reserve, Hadgarh Wildlife Sanctuary and

Kuldiha Wildlife Sanctuary.

A. 1, 3, and 4 oly

B. 1, 2 and 3 only

C. 1, and 4 only

D. 1 , 2 3 and 4 only

Answer: D

Explanation

• Similipal, which derives its name from ‘Simul’ (silk cotton) tree, is a national park and a tiger reserve

situated in the northern part of Odisha’s Mayurbhanj district.

• Similipal and the adjoining areas, comprising 5,569 sq km, was declared a biosphere reserve by the

Government of India on June 22, 1994, and lies in the eastern end of the eastern ghat.

• It was declared a biosphere reserve by the Government of India in June, 1994 and has been part of

the UNESCO World Network of Biosphere Reserve since 2009.

Simlipal Biosphere Reserve

• It includes three protected areas — Similipal Tiger Reserve, Hadgarh Wildlife Sanctuary with 191.06

km2 (73.77 sq mi) and Kuldiha Wildlife Sanctuary.

• It is the abode of 94 species of orchids and about 3,000 species of plants.

• The identified species of fauna include 12 species of amphibians, 29 species of reptiles, 264 species of

birds and 42 species of mammals, all of which collectively highlight the biodiversity richness of

Similipal.

• Sal is a dominant tree species.

• Similipal and the adjoining areas, comprising 5,569 sq km, was declared a biosphere reserve by the

Government of India on June 22, 1994, and lies in the eastern end of the eastern ghat.

• This protected area is part of the UNESCO World Network of Biosphere Reserves since 2009.

• Eucalyptus trees planted recently in Similipal Tiger Reserve (STR) and other forests in Odisha may be

among the reasons the reserve continues to be on fire 15 days after the first reported incident,

according to experts.

✓Eucalyptus trees, which are prone to fire, were planted in large tracts of the forest by clearing

medicinal plants and other native trees

66Available on App store

and Play Storewww.sleepyclasses.com

Call 6280133177

T.me/SleepyClasses

✓The leaves of these contain a highly inflammatory oil that ignites easily. The trees catch fire as the

ground beneath the trees is usually littered with leaves.

✓It is illegal for the forest department to plant eucalyptus trees instead of native ones such as sal,

mahuli, asan, karang, arjun, jack-fruits and other trees for which the forest fire is spreading.

✓The state forest department started planting eucalyptus trees in Joshipur and Kaling forest areas

of Similipal for the first time in 1977.

13.The term Arribada is associated with which of the following ?

A. Hatching

B. Fungi

C. Photosynthesis

D. Kraits

Answer: A

Explanation

• Olive Ridley female turtles have started moving to the Gahirmatha beach in the Kendrapara district in

Odisha.

• They are arriving at the beach to lay eggs which is described as the phenomenon called ‘arribada’.

• Arribada is a Spanish term which describes the unique natural heritage of these marine species which

converge on the nesting ground to lay eggs.

• The annual mass nesting of the marine species started on March 10, 2021 in Nasi-2 beach of

Gahirmatha nesting ground.

• Around 2,000 turtles had turned up to lay eggs on ground. The mass nesting process will continue for

at least 10 days.

• Usually, around 7.30 lakh Olive Ridley turtles were gathered for the mass nesting in the year

2019-2020.

Gahirmatha Beach

• It is a beach located in the state of Odisha.

• It separates the Bhitarkanika Mangroves from Bay of Bengal.

• This beach is the most important nesting beach for olive ridley sea turtles in the world.

• It is a part of the Gahirmatha Marine Wildlife Sanctuary. It is the only marine wildlife sanctuary in

Odisha.

Olive Ridley Sea Turtle

• The scientific name of the turtle is Lepidochelys olivacea.

• It is also known as the Pacific Ridley sea turtle.

www.sleepyclasses.com

Call 6280133177Available on App store

and Play Store 67

T.me/SleepyClasses

• It is the second-smallest known species across the world.

• The species is also the most abundant of all the sea turtles across the world.

• It is found in the warm and tropical waters especially in the Pacific and Indian Oceans. It is also found

in the warm waters of Atlantic Ocean.

• Olive Ridley turtle and the Kemp’s ridley turtle are known for their unique mass nesting sites which

are called as arribadas.

Kemp’s Ridley sea turtle

• It is also called Atlantic Ridley sea turtle.

• The turtle is the rarest species of sea turtle and also the most endangered species of sea turtles across

the world.

14.Three of the following criteria have contributed to the recognition of Western Ghats- Sri Lanka and

Indo-Burma regions as hotspots of biodiversity: (UPSC- 2011)

1. Species richness

2. Vegetation density

3. Endemism

4. Ethno-botanical importance

5. Threat perception

6. Adaptation of flora and fauna to warm and humid conditions

Which three of the above are correct criteria in this context?

A. 1, 2 and 6

B. 2, 4 and 6

C. 1, 3 and 5

D. 3, 4 and 6

Answer: C

Explanation

• Biodiversity is referred to as the variation of plant and animal species in a particular habitat. Species

evenness and species richness form the major components of biodiversity.

• India is known for its rich biodiversity and has around 24.46% of the geographical area covered by

forests and trees.

• Coined by Norman Myers, the term “Biodiversity hotspots” can be defined as the regions which are

known for their high species richness and endemism.

• According to Conservation International, a region must fulfil the following two criteria to qualify as a

hotspot:

68Available on App store

and Play Storewww.sleepyclasses.com

Call 6280133177

T.me/SleepyClasses

✓The region should have at least 1500 species of vascular plants i.e., it should have a high degree of

endemism.

✓It must contain 30% (or less) of its original habitat, i.e. it must be threatened.

• Following the criteria must for an area to be declared as Biodiversity Hotspot, there are major four

biodiversity hotspots in India:

✓The Himalayas

✓Indo-Burma Region

✓The Western Ghats

✓Sundaland

15.Abujmarh, a potential site for Biosphere Reserves as selected by Ministry of Forests and

Environment lies in

A. Maharashtra

B. Jharkhand

C. Chattisgarh

D. UttarPradesh

Answer: C

Explanation

• Abujmarh is a hilly forest area, spread over 4,000

square kilometres (1,500 sq mi) in Chhattisgarh,

covering Narayanpur district, Bijapur district and

Dantewada district.

• It is home to indigenous tribes of India, including

Gond, Muria, Abuj Maria, and Halbaas.

• It was only in 2009 that the Government of

Chhattisgarh lifted the restriction on the entry of

common people in the area imposed in the early

1980s.

• Geographically isolated and largely inaccessible,

the area continues to show no physical presence

of the civil administration, and is also known as

"liberated- zone" as it is an alleged hub of

Naxalite-Maoist insurgency, the banned

Communist Party of India (Maoist) and its military

wing, People's Liberation Guerilla Army (PLGA), who run a parallel government in the area

• In 2007 the area was proposed as a biosphere reserve by Ministry of Environment and Forests,

Government of India.

www.sleepyclasses.com

Call 6280133177Available on App store

and Play Store 69

T.me/SleepyClasses

16.Which of the following present day states are said to have been crossed by the mythical Sarasvati

river?

1. Punjab

2. Haryana

3. Rajasthan

4. Gujarat.

A. 1, 2 and 3 only

B. 2, 3 and 4 only

C.  1 and 2 only

D. 1, 2, 3 and 4

Answer: D

Explanation

• The Centre has reconstituted an advisory committee to chalk out a plan for studying the mythical

Sarasvati river for the next two years, after the earlier panel’s term ended in 2019.

• The ASI had first set up the committee on December 28, 2017 for a period of two years.

• The committee would be chaired by the Culture Minister and include officials from the Culture,

Tourism, Water Resources, Environment and Forest, Housing and Urban Affairs Ministries;

representatives of the Indian Space Research Organisation; officials from the governments of

Gujarat, Haryana and Rajasthan; and an ASI official.

• The river, which had originated from Kapal tirith in the Himalayas in the west of Kailash, was flowing

southward to Mansarovar and then taking a turn towards west.

• The river flowed through Haryana, Rajasthan and North Gujarat.

• It also flowed through Pakistan before meeting Western Sea through Rann of Kutch and was

approximately 4,000 km in length.

• The river had two branches: western and eastern.

• The Himalayan-born Satluj “of the PAST”, which flowed through the channels of present- day

Ghaggar-Patialiwali rivulets, represents the western branch of the ancient river. On the other hand,

Markanda and Sarsuti represented the eastern branch of Saraswati, known as Tons-Yamuna.

• The confluence of the branches was near Shatrana, 25 km south of Patiala. And suddenly, it flows

crossing the dessert (Rann of Kutch) and meet gulf of western sea.

• The Sarasvati River is one of the main Rigvedic rivers mentioned in the scripture Rig Veda and later

Vedic and post-Vedic texts.

• Book 6 of the Rig Veda includes a hymn called the ‘Nadistuti Sukta’, which sings praises of the

Saraswati as being “perfect mother, unsurpassed river, supreme goddess”.

• For 2000 years, between 6000 and 4000 B.C., the Saraswati flowed as a great river. "Sarasvati" may

also be identified with the Helmand or Haraxvati river in southern Afghanistan.

70Available on App store

and Play Storewww.sleepyclasses.com

Call 6280133177

T.me/SleepyClasses

17.World’s Largest Floating Solar Farms is being built in

A. Singapore

B. India

C. China

D. Rawanda

Answer: A

Explanation

• The largest floating solar farm of the world is being built in Singapore.

• The country has resorted to set up the energy plants off the coasts and reservoirs across it.

• This floating solar farm is being set up in the light that despite being one of the smallest countries

across the world, it is one of the biggest per capita carbon dioxide emitters in Asia.

• Thus, to cater to the issue of climate change and to cut the greenhouse gas emission it is building the

floating solar farm.

• The renewable energy is a challenge for Singapore because the country has no rivers for hydro-

electricity.

• The wind is also not that strong to power the turbines.

• Thus, with the setting up of the floating solar farms the authorities seek to change that notion.So, this

tropical country has turned to the solar power.

• Since, it has little land space the country has decided to setting up energy plants off its coasts and on

reservoirs.

• The island nation is threatened by the rising sea levels as a result of the climate change.

• So, the country is aware of urgency to cut down the emissions.

• In the light, the government of Singapore unveiled several “green plans” including the steps like

building more charging points to encourage the electric cars uses, reducing the amount of waste sent

to landfills and planting more trees.

• The country has also target to increase its solar energy use by four times amounting to two percent of

the power needs of the country by 2025 and then to three percent by 2030.

18.Shetrunjay Hills Reserve Forest Area lies in which of the following states

A. Maharashtra

B. Gujarat

C. Rajasthan

D. Goa

Answer: B

www.sleepyclasses.com

Call 6280133177Available on App store

and Play Store 71

T.me/SleepyClasses

Explanation

• Recently, a fire broke in the Shetrunjay Hills reserve forest area of Gujarat.

• The forest area is the habitat of Asiatic lions in the Bhavnagar territorial forest division.

• Shatrunjaya hills are located in Palitana city of Bhavnagar district in Gujarat. The hills are located on

the banks of river Shetrunji and are considered as a sacred hill by Jains.

• It has an altitude of164 feet above sea level.

• These hills are similar to other hills where Jain temples have been built in Gwalior, Mount Abu, Bihar

and Girnar.

• The hills are bounded by the Gulf of Khambhat in the south and Bhavnagar town in the north.

• The Jain’s sacred hill of Shatrunjaya comprises of 865 temples on it.

• These hills were sanctified when Rishabha gave his first sermon atop the hill.

• Rishabha was the first tirthankara of Jainism.

• The ancient history related to the hills can also be traced to Pundarika Swami who was a chief

Ganadhara and grandson of Rishabha.

• Swami had attained Nirvana or Moksha on the hill. Shetrunji River is an eastward-flowing river in

Gujarat. The river rises in the northeast of Gir Hills.

• The river has a maximum length of 227 kilometres.

19.Which of the following statements are correct about Mullaperiyar Dam ?

1. Constructed between 1887 and 1895, the dam redirected the river to flow towards the Bay of

Bengal, instead of the Arabian Sea and provide water to the arid rain region of Madurai in Madras

Presidency.

2. It is operated by Tamil Nadu

3. The dam is located on the confluence of the Mullayar and Periyar rivers

A. 1 and 2 only

B. 2 and 3 only

C. 1 and 3 only

D. 1, 2 and 3

Answer: D

Explanation

• The Supreme Court granted the liberty to serve on the Standing Counsel for the State of Kerala

notice, in a plea filed seeking termination of lease deed on State of Tamil Nadu and initiate necessary

steps for construction of 'protection wall' to contain damage that may be caused due to any distress

to the dam.

72Available on App store

and Play Storewww.sleepyclasses.com

Call 6280133177

T.me/SleepyClasses

• Although the dam is located in Kerala, it is operated by Tamil Nadu following an 1886 lease indenture

for 999 years (the Periyar Lake Lease Agreement) that was signed between the Maharaja of

Travancore and the Secretary of State for India for the Periyar Irrigation works.

• Constructed between 1887 and 1895, the dam redirected the river to flow towards the Bay of

Bengal, instead of the Arabian Sea and provide water to the arid rain region of Madurai in Madras

Presidency.

• The dam is located on the confluence of the Mullayar and Periyar rivers in Kerala’s Idukki district.

• The lease agreement was renewed in the 1970s by both Tamil Nadu and Kerala giving the former

rights to the land and water from the dam, besides the authority to develop hydropower projects at

the site. In return, Kerala would receive rent from Tamil Nadu.

• The first cracks in this agreement surfaced in 1979 when a minor earthquake had resulted in cracks in

the dam.

• The Central Water Commission, under the Government of India, conducted a study and

recommended lowering the water stored in the dam’s reservoir to 136 feet from 142 feet.

• If definitive measures were implemented, only then could the Tamil Nadu administration raise water

levels to the dam’s full capacity of 152 feet.

• Tamil Nadu claims that although it has undertaken measures to strengthen the dam, the Kerala

government has blocked any attempt to raise the reservoir water level – resulting in losses for

Madurai farmers.

• Kerala, however, highlights fears of devastation by residents living downstream in the earthquake-

prone district of Idukki.

• Scientists have argued that if there is an earthquake in the region measuring above six on the Richter

scale, the lives of over three million people will come under grave danger.

Supreme Court verdict

• In 2006, the Supreme court gave Tamil Nadu legal sanction to raise the water level to 142 feet.

• In response, Kerala amended the 2003 Kerala Irrigation and Water Conservation Act, restricting the

water level to 136 feet.

• In 2012, however, an Apex court-appointed committee stated that the dam was “structurally and

hydrologically safe” and that the Tamil Nadu government could raise water levels up to 142 feet.

• In 2014, the court event struck down the amendment to the 2003 Kerala Irrigation and Water

Conservation Act, calling it unconstitutional.

• The Supreme Court had also directed the Centre and the governments of Kerala and Tamil Nadu to

set up three panels to prepare a contingency plan in case of a disaster.

20.The hardy species which are the first to colonize barren environments or previously biodiverse

steady-state ecosystems that have been disrupted, such as by fire are known as

A. PrimarySpecies

www.sleepyclasses.com

Call 6280133177Available on App store

and Play Store 73

T.me/SleepyClasses

B. SecondarySpecies

C. Pioneer Species

D. Climax Species

Answer: C

Explanation

• previously biodiverse steady-state ecosystems that have been disrupted, such as by fire.

• Some lichens grow on rocks without soil, so may be among the first of life forms, and break down the

rocks into soil for plants.

• Since some uncolonized land may have thin, poor quality soils with few nutrients, pioneer species are

often hardy plants with adaptations such as long roots, root nodes

containing nitrogen-fixing bacteria, and leaves that employ transpiration.

• The plants that are often pioneer species also tend to be wind-pollinated rather than insect-

pollinated, as insects are unlikely to be present in the usually barren conditions in which pioneer

species grow; however, pioneer species tend to reproduce asexually altogether, as the extreme or

barren conditions present make it more favourable to reproduce asexually in order to increase

reproductive success rather than invest energy into sexual reproduction.

• Pioneer species will die creating plant litter, and break down as "leaf mold" after some time, making

new soil for secondary succession (see below), and nutrients for small fish and aquatic plants in

adjacent bodies of water.

74Available on App store

and Play Storewww.sleepyclasses.com

Call 6280133177

T.me/SleepyClasses

6. Science & Technology To watch the following questions on YouTube, click on the links given below

• Video 1

• Video 2

• Video 3

• Video 4

1. Which of the following is the correct focus of study and space organisation responsible for carrying

out the PUNCH Mission?

A. Asteroid Bennu, NASA

B. Asteroid Ryugu, JAXA

C. Mars, ISRO

D. Sun, NASA

Answer: D

Explanation

• PUNCH stands for Polarimeter to Unify the Corona and Heliosphere

• NASA’s PUNCH Mission is focussed on understanding the transition of particles from the Sun’s outer

corona to the solar wind that fills interplanetary space

• It consists of a constellation of four suitcase-sized microsats that will orbit the Earth in a formation

• It is expected to be launched in 2022.

2. Which of the following are incorrectly matched

1. Tri-Netra : Satellite

2. Project Netra : Debris tracking

3. Sindhu Netra : Railways

A. 1 only

B. 2 only

C. 1 and 3 only

D. 1, 2 and 3

Answer: C

Explanation

www.sleepyclasses.com

Call 6280133177Available on App store

and Play Store 75

T.me/SleepyClasses

• TRI-NETRA (Terrain imaging for diesel dRivers INfra-red, Enhanced opTical & Radar Assisted system):

Ministry of Railways initiated a proposal to install TRI- NETRA systems on locomotives for enhancing

the vision of Locomotive Pilots in inclement weather.

• Sindhu Netra is a satellite of DRDO to keep an eye on the ships in the Indian Ocean.

3. Epidemiology is the study of how diseases spread. In this context what or who is patient zero or

index case?

A. First documented person with the disease in the investigation

B. Person from whom the disease has not spread further

C. First casualty due to the spread of the disease

D. None of the above

Answer: A

Explanation

• Index case is the first case of a condition or syndrome (not necessarily contagious) to be described in

the medical literature whether or not the patient is thought tone the first person affected.

• It may not indicate the source of the disease, the possible spread or which reservoir holds the disease

in between outbreaks, but may bring awareness of an emerging outbreak.

4. Which of the following is not a feature of Cannabis plants?

A. They are not pollinated by bees and insects

B. It is used to produce industrial hemp products

C. Male and female flowers are found on different plants

D. All of the above

Answer: A

Explanation

• It is a genus of medicinal, recreational and fibre plant

• Cannabis along with plants like Coconut Palm, Date Palm, Maize etc. are pollinated by Wind

(Anemophily).

5. Helina and Dhruvastra fall in the category of

A. Air-to-air missiles

B. Surface-to-air missiles

C. Supersonic missiles

76Available on App store

and Play Storewww.sleepyclasses.com

Call 6280133177

T.me/SleepyClasses

D. Anti-tank guided missiles

Answer: D

Explanation

• Helina (the Army version) and Dhruvastra (IAF version) are launched from Advanced •Light

Helicopter (ALH) platform.

• The system has all-weather day-and-night capability and can defeat battle tanks with conventional

armour as well as with explosive reactive armour.

• It is one of the most-advanced anti-tank weapons in the world.

6. Bhimbetka caves near Bhopal were in news recently for a fossil dating back about 550 million years.

The fossil was of

A. Spriggina

B. Dickinsonia

C. Pikaia

D. Wiwaxia

Answer: B

Explanation

• Dickinsonia are the earliest known living animals.

• They belonged to the Ediacaran period of the earth’s history named after Ediacara hills in South

Australia.

7. Spitzer Space telescope, one of NASA’s four Great Observatories, studied the universe by detecting

A. X-rays

B. Gamma rays

C. Visible light

D. Infrared rays

Answer: D

Explanation

• NASA decommissioned the Spitzer Space Telescope which studied the universe by detecting cosmic

infrared radiations.

• It was launched in 2003, was one of NASA's four Great Observatories, along with the Hubble Space

Telescope, the Chandra X-ray Observatory and the Compton Gamma Ray Observatory.

www.sleepyclasses.com

Call 6280133177Available on App store

and Play Store 77

T.me/SleepyClasses

8. Which of the following are true regarding NISAR

1. It is a joint collaboration of NASA and European Space Agency for producing extremely high-

resolution images for earth observation

2. NASA is providing the mission’s high-rate communication subsystem and GPS receivers along

with other equipment

A. 1 only

B. 2 only

C. Both 1 and 2

D. Neither 1 nor 2

Answer: B

Explanation

• ISRO has completed development of Synthetic Aperture Radar (SAR) capable of •producing

extremely high resolution images for a joint earth observation satellite mission with NASA

• NASA-ISRO SAR (NISAR) is a joint collaboration for a dual-frequency L and S-band SAR for earth

observation

• It will be the first satellite mission to use two different radar frequencies (L-band and S- band) to

measure changes in our planet’s surface less than a centimetre across

• NASA is providing the mission’s L-band SAR, a high-rate communication subsystem for science data,

GPS receivers, a solid-state recorder and payload data subsystem.

• ISRO is providing the spacecraft bus, the S-band radar, the launch vehicle and •associated launch

services for the mission, whose goal is to make global measurements of the causes and consequences

of land surface changes using advanced radar imaging

• Click here to learn more

9. Which of the following correctly defines a Super-Earth?

A. Exoplanet larger than Earth yet lighter than ice giants such as Neptune and Uranus

B. They can be made of gas, rock or a combination of both

C. Both A and B

D. Neither A nor B

Answer: C

Explanation

• Super-Earths – a class of planets unlike any in our solar system – are more massive •than Earth yet

lighter than ice giants like Neptune and Uranus, and can be made of gas, rock or a combination of both

• They are between twice the size of Earth and up to 10 times its mass

78Available on App store

and Play Storewww.sleepyclasses.com

Call 6280133177

T.me/SleepyClasses

• Super-Earth is a reference only to an exoplanet’s size – larger than Earth and smaller than Neptune –

but not suggesting they are necessarily similar to our home planet.

10.The terms ‘Predator-B’, ‘SeaGuardian’ refer to ‘MQ-9B’,

A. Fighter jets

B. Tanks

C. Drones

D. Ships

Answer: C

11.AsterX recently heard in news is a

A. Military Satellite

B. Space Military Exercise

C. Fighter Aircraft

D. New found layer of Earth

Answer: B

Explanation

• Amid the growing competition among the world powers in the field of space, France has begun its first

space military exercises to test its ability to defend its satellites.

• The exercise drills are codenamed “AsterX” to commemorate the first French satellite Asterix from

1965.

• The exercises are part of France’s strategy to become the world’s third-largest space power, after the

USA and China.

• These space military exercises are the first ever attempt not only for the French army but also for

Europe.

• Moreover, France has planned to develop anti-satellite laser weapons and to strengthen surveillance

capabilities to close the gap with rivals; China and Russia.

• Aim of the Exercise is to monitor a potentially dangerous space object as well as threats to its own

satellite from another foreign power possessing a considerable space force.

• Along with France, the new US Space Force and German space agencies are taking part in the

exercises.

12.Which of the following statements are correct about ‘Make-II’ category Projects?

1. They comprise of the prototype development of system, equipment, platform or their upgrades.

www.sleepyclasses.com

Call 6280133177Available on App store

and Play Store 79

T.me/SleepyClasses

2. Such projects are ‘Industry Funded’.

3. This category of the project provides for a focus on the impor substitution and innovative

solutions.

A. 1 and 2 only

B. 2 and 3 only

C. 1 and 3 only

D. 1, 2 and 3

Answer: D

Explanation

• Context: The Indian army is all set to procure Made in India “Mobile Integrated Network Terminal

(MINT)” systems providing a huge boost to the Aatmanirbhar Bharat Abhiyaan of the government.

• The MINT system will enhance the operational communication capability of the Indian Army in

accordance with the Atmanirbhar Bharat Abhiyaan.

• MINT systems come under the Make II Category of DAP 2020.

• This system is a portable, lightweight, state of art integrated communication solution comprising of

the satellite backhaul and wireless access system to provide the support voice, video and data.

• The ‘Make-II’ category projects which comprises of the prototype development of system, equipment,

platform or their upgrades.

• Such projects are ‘Industry Funded’.

• The projects could also comprise of the sub-systems or sub- assembly or assemblies or components of

the developing systems.

• This category of the project provides for a focus on the import substitution and innovative solutions.

• For prototype development purposes under this category, no government funding is provided.

• As per the evaluation of response submitted by Indian Industry total of eleven firms were issued the

Project Sanction Order on March, 12 2021.

• These firms will develop the prototype.

• Following this, Contract will be placed with one of the firms after they successfully develop the

prototype in accordance with the provisions of Buy (Indian-IDDM) of DAP 2020.

13.Non-fungible token recently heard in news is used in the context of

A. Crypto Currency

B. Covid-19

C. Digital assets

D. Digital Penalties

80Available on App store

and Play Storewww.sleepyclasses.com

Call 6280133177

T.me/SleepyClasses

Answer: C

Explanation

• Context: An American rock band recently released a music album

• The music album was launched as a non-fungible token (NFT).

• It is a special type of cryptographic token which represents somethings which are unique.

• These tokens are not mutually interchangeable.

• This is opposite to the cryptocurrencies such as bitcoin.

• It is a type of crypto-asset that represents several numbers of assets such as tickets, artforms, and

music.

• Such type of asset is gaining its popularity because several investors and enthusiasts are interested to

spend a large sum of money on items which only exist digitally.

• These tokens are used to create digital scarcity and digital ownership.

• It can also be used in online gaming, crypto art and digital collectibles.

14.Based on the following statements identify the personality

1. He is remembered as the “India’s Satellite Man”.

2. He also worked as the chairman of IndianS pace Research Organisation.

3. The tech giant Google celebrated the 89th birthday of this renowned Indian professor and

scientist on March 10, 2021 through its “Google Doodle”.

4. He was awarded with the Padma Bhushan by Government of India in the year 1976, and he was

conferred with the Padma Vibhushan in the 2017

5. He was inducted into Satellite Hall of Fame, Washington in March 2013 at the ceremony

organised by the Society of Satellite Professionals International

6. The professor was also inducted in International Astronautics Federation (IAF) in May 2016 and

became the first Indian again to achieve this honour.

A. Udupi Ramachandra Rao

B. K. Sivan

C. Satish Dhawan

D. Homi J. Bhabha

Answer: A

Explanation

• Udupi Ramachandra Rao was an Indian space scientist.

• He also worked as the chairman of Indian Space Research Organisation.

www.sleepyclasses.com

Call 6280133177Available on App store

and Play Store 81

T.me/SleepyClasses

• The scientist was also the Chairman of Governing Council of Indian Institute for Space Science and

Technology (IIST) at Thiruvananthapuram, Physical Research Laboratory at Ahmedabad and Nehru

Planetarium at Bengaluru.

• He is remembered as the “India’s Satellite Man”.

• He died in the year 2017.

• He was born in a village of Karnataka on March 10, 1932.

• He started his career as a cosmic-ray physicist and as a protege of Dr Vikram Sarabhai who is

regarded as the father of India’s space program.

• He then worked as a professor and conducted experiments on Pioneer and Explorer space probes of

NASA.

• He returned to India in 1966 and lead the 1975 launch of India’s first satellite called “Aryabhata”.

• This satellite is one among 20 satellites the professor developed. The satellite transformed the rural

India by providing advanced communication and meteorological services.

• Professor Rao was awarded with the Padma Bhushan by Government of India in the year 1976, and

he was conferred with the Padma Vibhushan in the 2017.

• Professor was inducted into Satellite Hall of Fame, Washington in March 2013 at the ceremony

organised by the Society of Satellite Professionals International.

• Thus, he became the first Indian to be inducted into Satellite Hall of Fame. The professor was also

inducted in International Astronautics Federation (IAF) in May 2016 and became the first Indian

again to achieve this honour.

15.DUSTLIK II’ is a joint exercise between India and

A. Russia

B. Kyrgyzstan

C. Uzbekistan

D. Tajakistan

Answer: C

Explanation

• The India–Uzbekistan joint military exercise “DUSTLIK II” was conducted in the Foreign Training

Node Chaubatia at Ranikhet in the state of Uttarakhand.

• It was the second edition of annual bilateral joint exercise of both the armies.

• It concluded on March 19, 2021

• The first edition of DUSTLIK was held at Uzbekistan in the month of November, 2019.

• In the second edition of the exercise, around 45 soldiers from each of Indian army and Uzbekistan

army participated.

82Available on App store

and Play Storewww.sleepyclasses.com

Call 6280133177

T.me/SleepyClasses

• During the exercise, both the armies shared their expertise and skills with respect to the counter-

terrorism operations under the mandate of United Nations in Mountainous or Urban or Rural

scenarios.

• The exercise also culminated into a 36-hour joint validation exercise which was scheduled for March

17 to March 18, 2021.

• This exercise was a testbed for soldiers of both the countries because under the exercise, the armies

underwent the challenges of actual operations in such scenarios.

• The joint exercise provided an impetus to the military and diplomatic ties of the two countries.

• It also signified the strong resolve of both the countries to counter-terrorism.

16.Based on the following statements identify the INS.

1. It is an amphibious transport dock which presently serves the Indian Navy.

2. This ship was procured from the United States for US$ 90 million in the year 2005.

3. The ship then was commissioned to the service in June 2007.

4. This is the only Indian naval ship that has been procured from United States.

5. It is based in Visakha patnam under Eastern Naval Command.

A. INS Gaj

B. INS Jalashwa

C. INS Shalki

D. INS Shankul

Answer: B

Explanation

• Context: The Indian Naval Ship Jalashwa has reached the port of Anjouan in Comoros

• The ship was carrying a consignment of 1000 metric tonnes of rice.

• This consignment of 1000 metric tonnes of rice as food assistance were flagged off to Comoros in the

light of promise made by the Vice President of India, Venkaiah Naidu during his brief visit to Comoros

in the month of October 2019.

• After handing over the consignment to Comoros, the INS Jalashwa will travel to the Port of Ehoala so

as to deliver another consignment of 1000 metric tonne of rice and 1,00,000 tablets of HCQ.

• This consignment will be delivered in accordance with the promise of External Affairs Minister.

• He made his promise after the Madagascar asked for the solidarity and assistance from the Indian

side to deal with the humanitarian crisis in South of Madagascar because of severe drought.

• The ship will also deploy the Indian Naval Training Team in Madagascar for capacity building and

training of the Malagasy Special Forces for two weeks.

www.sleepyclasses.com

Call 6280133177Available on App store

and Play Store 83

T.me/SleepyClasses

INS Jalashwa

• It is an amphibious transport dock which presently serves the Indian Navy.

• This ship was procured from the United States for US $90 million in the year 2005.

• The ship then was commissioned to the service in June 2007.

• This is the only Indian naval ship that has been procured from United States.

• It is based in Visakhapatnam under Eastern Naval Command.

• Eastern Naval Command is one among the three command-level formations of Indian Navy.

• It is based in Visakhapatnam, Andhra Pradesh.

• This command level is responsible for all the naval forces in Bay of Bengal, parts of Indian Ocean and

naval establishments on east coast of India.

• This command was set up in March 1968.

• It is commanded by a Three Star Flag Officer of the rank of Vice Admiral.

• The vice admiral flag officer is titled as the Flag Officer Commanding-in-Chief Eastern Command.

17.Which of the following statements about Rare disease are incorrect?

1. They are also known as neglected tropical disease.

2. These are characterised by a wide diversity of symptoms and signs that vary not only from

disease to disease but also from patient to patient suffering from the same disease.

Select the correct code.

A. 1 only

B. 2only

C. Both 1 and 2

D. Neither 1 nor 2

Answer: A

Explanation

• A rare disease, also referred to as an orphan disease, is any disease that affects a small percentage of

the population.

• Most rare diseases are genetic, and are present throughout a person’s entire life, even if symptoms do

not immediately appear. In Europe a disease or disorder is defined as rare when it affects less than 1

in 2000 citizens.

• Rare diseases are characterised by a wide diversity of symptoms and signs that vary not only from

disease to disease but also from patient to patient suffering from the same disease. Relatively

common symptoms can hide underlying rare diseases, leading to misdiagnosis.

84Available on App store

and Play Storewww.sleepyclasses.com

Call 6280133177

T.me/SleepyClasses

• The most common rare diseases recorded in India are Haemophilia, Thalassemia, sickle-cell anaemia

and primary immuno deficiency in children, auto-immune diseases, Lysosomal storage disorders such

as Pompe disease, Hirschsprung disease, Gaucher’s disease, Cystic Fibrosis, Hemangiomas and

certain forms of muscular dystrophies.

• India does not have a definition of rare diseases because there is a lack of epidemiological data on its

incidence and prevalence.

• While there is no universally accepted definition of rare diseases, countries typically arrive at their

own descriptions, taking into consideration disease prevalence, its severity and the existence of

alternative therapeutic options.

• In the US, for instance, a rare disease is defined as a condition that affects fewer than 200,000 people.

• The same definition is used by the National Organisation for Rare Disorders (NORD) in India.

18.Which of the following statements about Helium are correct?

1. It is a colourless, odourless, tasteless, non-toxic, inert, monatomic-gas.

2. It is the first in the noble gas group in the periodic table.

3. Its boiling point is the lowest among all the elements.

4. India’s Rajmahal volcanic basin is the store house of helium

Select the correct code.

A. 1 and 2 only

B. 2, 3 and 4only

C. 1 and 4 only

D. 1, 2, 3 and 4

Answer: D

Explanation

• Context: India imports helium for its needs and with the U.S. appearing set to cut off exports of helium

since 2021, the Indian industry stands to lose out heavily.

• Helium is not just for balloons but it is the key ingredient for India’s high technology and the most

sophisticated medical diagnosis

• Helium is a chemical element with the symbol He and atomic number 2.

• It is a colourless, odourless, tasteless, non- toxic, inert, monatomic gas, the first in the noble gas group

in the periodic table. Its boiling point is the lowest among all the elements.

Helium in India

• India’s Rajmahal volcanic basin is the storehouse of helium trapped for billions of years, since the very

birth of our Earth from the Sun.

www.sleepyclasses.com

Call 6280133177Available on App store

and Play Store 85

T.me/SleepyClasses

• At present, researchers are mapping the Rajmahal basin extensively for future exploration and

harnessing of helium.

Why India needs Helium

• Every year, India imports helium worth Rs 55,000 crores from the U.S. to meet its needs.

• Helium is used in medicine, scientific research, for blimp inflation, party balloons as well as having

welding applications.

• It finds many applications, mainly in magnetic resonance imaging (MRI) scans, in rockets and in

nuclear reactors.

19.Jhuran formation is related to which of the following

A. Coal

B. Limestone

C. Corals

D. Iron

Answer: D

Explanation

• Context: In 2004, NASA’s Mars

exploration rover ‘Opportunity’ found

several small spheres on the planet,

informally named Martian blueberries

which find a resemblance to the similar

formation in India’s Kutch region.

Martian blueberries

• Opportunity’s mini spectrometers

studied mineralogy and noted they were made of iron oxide compounds called haematites.

• This caused excitement, as the presence of haematites suggests that there was water present on

Mars.

• The widely accepted formation mechanism of hematite concretion [hard solid mass] is precipitation

from aqueous fluids.

• Hematite is known to form in oxidizing environments hence it can be inferred that water must have

played a crucial role in the formation of grey hematite on Mars.

• Indian researchers have been studying hematite concretions in Kutch called the Jhuran formation.

• These formations are 145 and 201 million years old.

• Detailed geochemistry and spectroscopic investigations of the haematite concretions in this area

revealed that they resemble the ones on Mars.

86Available on App store

and Play Storewww.sleepyclasses.com

Call 6280133177

T.me/SleepyClasses

• They have similar morphology – spherical, often doublet and triplet – and similar mineralogy – a

mixture of haematite and goethite.

• Hence, several types of research have shown that the Kutch area is a potential Martian analogue

locality.

20.Which of the following statements about sounding Rockets are incorrect?

1. These are one or two-stage solid propellant rockets used for probing the upper atmospheric

regions and for space research.

2. The sounding rockets generally stay between 400 km and 1600 kilometres in the atmosphere.

Select the correct code.

A. 1 only

B. 2only

C. Both 1 and 2

D. Neither 1 nor 2

Answer: B

Explanation

• Context: ISRO launched sounding rocket RH-560, to study neutral winds, plasma dynamics

• It is also called a research rocket or rocket sonde.

• The sounding rockets are one or two stage solid propellant rockets used to study upper atmospheric

regions and for space research.

• They serve as easily affordable platforms that test prototypes of new components or subsystems

intended for use in launch vehicles and satellites.

• The sounding rockets generally stay between 40 km and 160 kilometres in the atmosphere.

• The range is such because the weather balloons have to stay within 40 kilometres and the Space

rockets.

• The rockets are placed above it.

• In case the Sounding rockets go above 160 km, they may burst.

• ISRO began with the launch of indigenously built sounding rockets from 1965.

• Its experience gained was valuable in gaining excellence in the solid propellant technology.

• In 1975, all sounding rocket activities were cluttered under the Rohini Sounding Rocket (RSR)

Programme. RH-75, with a diameter of 75 mm was the first truly Indian sounding rocket, which was

followed by RH-100 and RH-125 rockets.

• The series of sounding rockets are called Rohini series with RH 200, RH 300 and RH 560 being the

most important among them.

• The numbers along with the name acronym stands for the diameter of the rocket in milimetres. www.sleepyclasses.com

Call 6280133177Available on App store

and Play Store 87

T.me/SleepyClasses

• The Thumba Equatorial Rocket Launching Station was established on 21st November 1963 and was

operated by ISRP.

• Its southern tip is close to earth's magnetic equator.

• It marked the beginning of Indian Space Programme and was the bedrock of the launch vehicles built.

• Before the indigenous efforts the first rockets were two-stage rockets imported from Russia (M-100)

and France (Centaure).

• At that time M-100 was able to carry a payload of 70 kg to an altitude of 85 km, the Centaure was

capable of reaching 150 km with a payload of approximately 30 kg.

88Available on App store

and Play Storewww.sleepyclasses.com

Call 6280133177